0% found this document useful (0 votes)
13 views

Sat Reading Book 2

The document is a practice book for the Digital SAT Reading section, containing various reading comprehension questions and passages. It includes excerpts and questions that assess understanding of themes, arguments, and vocabulary in context. The content is designed to help students prepare for the reading portion of the SAT exam.

Uploaded by

naominguyen247
Copyright
© © All Rights Reserved
We take content rights seriously. If you suspect this is your content, claim it here.
Available Formats
Download as PDF, TXT or read online on Scribd
0% found this document useful (0 votes)
13 views

Sat Reading Book 2

The document is a practice book for the Digital SAT Reading section, containing various reading comprehension questions and passages. It includes excerpts and questions that assess understanding of themes, arguments, and vocabulary in context. The content is designed to help students prepare for the reading portion of the SAT exam.

Uploaded by

naominguyen247
Copyright
© © All Rights Reserved
We take content rights seriously. If you suspect this is your content, claim it here.
Available Formats
Download as PDF, TXT or read online on Scribd
You are on page 1/ 71

HỌC DSAT CÙNG MS.

VÂN ANH

DIGITAL SAT READING


PRACTICE BOOK 1

1
https://hocsat.com/
NUMBER SAT DIGITAL READING QUESTIONS

"What's the matter with him?" Maneck asked his mother. "When I'm here, he ignores me or fights with me. When I'm at
1. school, he writes letters saying how much he misses me."
"You have to understand," said Mrs. Kohlah, "people change when times change. It does not mean he doesn't love you.
Compared with Maneck, Mrs. Kohlah is more
A) guarded about discussing Mr. Kohlah.
B) determined to appease Mr. Kohlah.
C) concerned about Mr. Kohlah's irritability,
D) accepting of Mr. Kohlah's behavior.
“I don’t think that you would want to have a lot of black hairs along the top of your back if you wanted to try to keep cool,”
2. said Tim Caro, a professor of wildlife biology at the University of California, Davis, who studies zebra stripes but was not
involved in the new study. “It’s kind of the last color that you would want.”
Caro would most likely agree with which of the following statements about coloration patterns containing black stripes?
A) They have an unknown genetic basis.
B) They might hinder thermoregulation in animals.
C) They occur in nature very rarely.
D) They are aesthetically unappealing.
3. “The only function of economic forecasting is to make astrology look respectable,” John Kenneth Galbraith, an irreverent
economist, once said. Since economic output represents the aggregated activity of billions of people, influenced by forces seen
and unseen, it is a wonder forecasters ever get it right. Yet economists cannot resist trying. As predictions for 2016 are
unveiled, it is worth assessing the soothsayers’ records.
The author uses the phrase "seen and unseen" most likely to
A) support the claim that economic forecasts rely on both explicit and unstated assumptions.
B) draw a distinction between tangible and intangible outputs of economic activity.
C) suggest that some factors affecting economic output are known while others are not.
D) allude to the two different approaches economists take when making forecasts.
4. “We are in a world that’s painted black and white,” says agricultural economist Matin Qaim. “In Europe in particular, people
are deeply convinced that genetically modified (GM) crops are bad for the world. If you say anything in favor of GM crops,
you are talking in favor of evil.”
That designation of evil is one of the two prevailing narratives concerning genetically engineered foods. Opponents of
genetically modified organisms (GMOs) tell the story that “Franker” organisms are a new technology that poses known and
unknowable dangers to human health, the environment, and society at large. On the other side, proponents argue that GMOs
are a harmless and necessary tool for saving a world threatened by overpopulation and a changing climate.
The first paragraph serves primarily to
A) dispute a long-standing scientific consensus.
B) introduce a problem that is specific to Europe.
C) draw attention to the controversial nature of GMOs.
D) express support for a worldwide ban on GMO agriculture.
“We’re realizing that the early Moon was a much more dynamic place than we thoughts," says Jeffrey Andrews-Hanna, a
5. planetary scientist at the Colorado School of Mines in Golden and lead author of a new study of the Procellarum’s geology.
The discovery also casts doubt on the decades—old theory that the circular Procellarum region is a basin, or giant crater,
created when a large asteroid slammed into the Moon. “We don’t expect a basin rim to have corners," Andrews—Hanna says.
Andrews-Hanna’s comment about a basin rim primarily serves to
A) point out how an observation conflicts with existing understanding.

2
https://hocsat.com/
B) refine the definition of a type of geological feature.
C) identify an analogy between the Moon’s geology and that of Earth.
D) indicate evidence of erroneous measurement results.
6. “What I love about the study is how it communicates something that memory-distortion researchers have suspected for some
time, that perhaps no one is immune to memory distortion,” said Patihis.
What the study doesn’t do, Patihis admits, is explain why highly superior autobiographical memory people exist at all. Their
prodigious recall is a matter of scientific fact, and one of the goals of the new work was to see if an innate resistance to
manufactured memories might be one of the reasons. But on that score, the researchers came up empty.
Which statement about the study led by Patihis can reasonably be inferred from the text?
A) Its overall goal has been questioned by several researchers.
B) Its main finding was not a surprise to certain scientists.
C) Its methodology is thought to be highly innovative.
D) It provided a definitive resolution to an ongoing debate.
7. 75 percent of subjects joined the group in giving the wrong answer - and save themselves the embarrassment of being odd man
out - in at least one round.
Now, it’s easy to dismiss the study participants as being too easily manipulated. But regardless of how independent-minded
and steadfast we may think we are, we’re all tempted at times to align ourselves with a group. We may worry that if we voice
an unpopular viewpoint others will doubt our intelligence, taste, or competence. Or we may just not want to make waves. The
challenge is to know when to speak up.
According to the text, the authors’ opinion of the study subjects who conformed with the group is best characterized as
A) disapproving.
B) ambivalent.
C) understanding.
D) concerned.
8. “And do I ask, wherefore my heart
Falters, oppressed with unknown needs?
Why some inexplicable smart
All movement of my life impedes?
Alas! In living Nature’s stead,
Where God His human creature set,
In smoke and mold the fleshless dead
And bones of beasts surround me yet!”
How is humanity contrasted with God in this passage?
A. God is the giver and man has destroyed the gift of life
B. As the source of life and the scientific investigator of life
C. They are not contrasted.
D. As truly living and nearly dead
9. A __________ example is the Hudson River, which has a deep channel extending far across the continental shelf. However, it
is well known that rivers can no longer erode a valley once they enter the sea—instead, they typically deposit sediments and
build up a delta. It was immediately clear that the now-submerged channels had once been above sea level. But exactly when
this erosion occurred was not known, and the precise amount of sea-level lowering was also difficult to determine with
certainty.
Which choice completes the text with the most logical and precise word or phrase?
A) representative.
B) timeless.

3
https://hocsat.com/
C) unrivaled.
D) outdated.
10. A __________ example is the Hudson River, which has a deep channel extending far across the continental shelf. However, it
is well known that rivers can no longer erode a valley once they enter the sea—instead, they typically deposit sediments and
build up a delta. It was immediately clear that the now-submerged channels had once been above sea level. But exactly when
this erosion occurred was not known, and the precise amount of sea-level lowering was also difficult to determine with
certainty.
Which choice completes the text with the most logical and precise word or phrase?
A) representative.
B) timeless.
C) unrivaled.
D) outdated.
11. A black hole is a region in space containing a mass so dense that not even light can escape its gravity. Black holes are
invisible, but astronomers can find them by __________ their gravitational pull on other objects. Matter being pulled into a
black hole gathers around it like storm debris circling a tornado’s center.
Which choice completes the text with the most logical and precise word or phrase?
A) trailing.
B) traversing.
C) monitoring.
D) paralleling.
12. A black hole is a region in space containing a mass so dense that not even light can escape its gravity. Black holes are
invisible, but astronomers can find them by __________ their gravitational pull on other objects. Matter being pulled into a
black hole gathers around it like storm debris circling a tornado’s center.
Which choice completes the text with the most logical and precise word or phrase?
A) trailing.
B) traversing.
C) monitoring.
D) paralleling.
13. A black hole is a region in space containing a mass so dense that not even light can escape its gravity. Black holes are
invisible, but astronomers can find them by tracking their gravitational pull on other objects. Matter being pulled into a black
hole gathers around it like storm debris circling a tornado’s center. As this cosmic stuff rubs together it produces friction and
light, making black holes among the universe’s brightest objects.
Which choice best states the main purpose of the text?
A) provide examples of black holes and discuss the properties of each.
B) describe the nature of black holes and explain how they can be detected.
C) present competing theories about the origins and characteristics of black holes.
D) illustrate the differences between supermassive and intermediate-mass black holes.
14. A business analysis of the Appenian railroad system divided its long-distance passenger routes into two categories: rural routes
and interurban routes. The analysis found that, unlike the interurban routes, few rural routes carried a high enough passenger
volume to be profitable. Closing unprofitable rural routes, however, will not necessarily enhance the profitability of the whole
system, since___________
Which of the following, if true, most logically complete the argument?
(A) a large part of the passenger volume on interurban routes is accounted for by passengers who begin or end their journeys
on rural routes
(B) within the last two decades several of the least used rural routes have been closed and their passenger services have been

4
https://hocsat.com/
replaced by buses
(C) the rural routes were all originally constructed at least one hundred years ago, whereas some of the interurban routes were
constructed recently for new high-speed express trains
(D) not all of Appenia's large cities are equally well served by interurban railroad services
15. A cart at the end of the sidewalk caught her eye. Hattie had never seen a flower vendor’s cart. A white man set on a stool with
his shirtsleeves rolled and his hat tipped forward against the sun. A Negro woman approached the cart. She indicated a bunch
of flowers. The white man stood—he did not hesitate, his body didn’t contort into a posture of menace -and took the flowers
from a bucket. Before wrapping them in paper, he shook the water gently from the steins. The Negro woman handed him the
money. As the woman took her change and moved to put it in her purse, she upset three of the flower arrangements. Vases and
blossoms tumbled from the cart and crashed on the pavement. Hattie stiffened, waiting for the inevitable explosion. She waited
for the other Negroes to step back and away from the object of the violence that was surely coming. She waited for the
moment in which she would have to shield her eyes from the woman and whatever horror would ensue. The vendor stooped to
pick up the mess. The Negro woman gestured apologetically and reached into her purse again, presumably to pay for what
shell damaged. In a couple of minutes it was all settled, and the woman walked on down the street with her rose in the paper
cone of flowers, as if nothing had happened.
Based on the text, how does Hattie believe many black people would react to a confrontation like the one she anticipates
between the flower vendor and his customer?
A) They would condemn the confrontation openly but lake care not to intervene in it.
B) They would distance themselves as much as possible from the confrontation.
C) They would defuse the confrontation through calming words and actions.
D) They would argue about the significance of the confrontation among themselves.
16. A child learning to play the piano will not succeed unless the child has an instrument at home on which to practice. However,
good quality pianos, whether new or secondhand, are costly. Buying one is justified only if the child has the necessary talent
and perseverance, which is precisely what one cannot know in advance. Consequently, parents should buy an inexpensive
secondhand instrument at first and upgrade if and when the child's ability and inclination are proven.
Which of the following, if true, casts the most serious doubt on the course of action recommended for parents?
A. Learners, particularly those with genuine musical talent, are apt to lose interest in the instrument if they have to play on a
piano that fails to produce a pleasing sound.
B. Reputable piano teachers do not accept children as pupils unless they know that the children can practice on a piano at
home.
C. ideally, the piano on which a child practices at home should be located in a room away from family activities going on at
the same time.
D. Very young beginners often make remarkable progress at playing the piano at first, but then appear to stand still for a
considerable period of time.
17. A great power has been placed in the hands of those who direct the advertising policies of our country, and power is always
coupled with responsibilities. No occupation is _________ with greater obligations than that which partakes of the nature of
education. Those engaged in that effort are changing the trend of human thought. They are molding the human mind.
Which choice completes the text with the most logical and precise word or phrase?
A) tasked.
B) blamed.
C) assaulted.
D) purchased.
18. A great power has been placed in the hands of those who direct the advertising policies of our country, and power is always
coupled with responsibilities. No occupation is _________ with greater obligations than that which partakes of the nature of
education. Those engaged in that effort are changing the trend of human thought. They are molding the human mind.

5
https://hocsat.com/
Which choice completes the text with the most logical and precise word or phrase?
A) tasked.
B) blamed.
C) assaulted.
D) purchased.
19. A great power has been placed in the hands of those who direct the advertising policies of our country, and power is always
coupled with responsibilities. No occupation is charged with greater obligations than that which partakes of the nature of
education. Those engaged in that effort are changing the trend of human thought. They are molding the human mind. Those
who write upon that tablet write for all eternity.
The text suggests that those who create advertisements are similar to
A) inventors who do not understand the full significance of their inventions.
B) officials who set policies that regulate the behavior of citizens.
C) teachers who permanently influence the ideas and beliefs of others.
D) artists who produce works of great beauty for people to admire.
20. A recent study by professor Georg Jander’s group at the Boyce Thompson Institute (BTI) finds that corn plants may make
serious trade-offs when defending themselves against multiple types of insects. Some corn varieties make themselves more
vulnerable to aphids after generating defensive compounds against nibbling caterpillars. The results may lead to the
development of corn plants that are naturally more resistant to certain insects.
Which situation is most similar to the one described in underlined lines?
A) A bird expends energy searching for food but glides to conserve energy whenever possible.
B) A tortoise develops a heavy shell for protection but sacrifices some mobility in the process.
C) A bear hibernates in the winter and loses minimal body mass during that time.
D) A wolf eats more than its share of food and leaves the rest of its pack hungry.
21. A short period of __________ felicity followed, and but a short one. Troubles soon arose. Sir Walter, on being applied to,
without actually withholding his consent, or saying it should never be, gave it all the negative of great astonishment, great
coldness, great silence, and a professed resolution of doing nothing for his daughter.
Which choice completes the text with the most logical and precise word or phrase?
A) intricate.
B) intense.
C) delicate.
D) elegant.
22. A study by a team of British researchers published recently in the journal POS ONE found a strong correlation between a
book's "literary misery index" (the frequency of words such as "anger," "disgust," fear," and "sadness") and the economic
misery index (a measure of unemployment and inflation) of either the U.S. or Britain for the ten years that preceded its
publication.
Of course, this correlation, whether in the U.S., Great Britain or Germany, might not come as a huge shock obviously, the
circimstances that surround an author influence his or her word choices. But the fact that the signal of economic times could be
consistently spotted through the noise of all of an author's personal circumstances is still somewhat surprising, and shows what
a profound effect economics have on our creative mindsets. As Bentley put it, "global economics is part of the shared
emotional experience of the 20th century."
The author anticipates which objection to the researchers' conclusion?
A) It seems self-evident.
B) It is derived from outdated methodology.
C) It has no real-world application.
D) It is based on insufficient data.

6
https://hocsat.com/
23. After her travels with Dr. Miller, Miss Spivey continued her education by attending Barnard College in New York City. She
told us all that at school the first day. When my little brother Ralphord asked what did she study at Barnyard College, Miss
Spivey explained that Barnard, which she wrote on the blackboard, was the sister school of Columbia University, of which,
she expected, we all had heard.
The interaction between Miss Spivey and Ralphord serves mainly to
A) suggest that Miss Spivey has an exaggerated view of what information should be considered common knowledge.
B) establish a friendly dynamic between the charming schoolchildren and their indulgent and doting new instructor.
C) introduce Ralphord as a precocious young student and Miss Spivey as a dismissive and disinterested teacher.
D) demonstrate that the children want to amuse Miss Spivey with their questions.
24. Although Ackerburg's subway system is currently operating at a deficit, the transit authority will lower subway fares next year.
The authority projects that the lower fares will result in a ten percent increase in the number of subway riders. Since the
additional income from the larger ridership will more than offset the decrease due to lower fares, the transit authority actually
expects the fare reduction to reduce or eliminate the subway system's operating deficit for next year.
Which of the following, if true, provides the most support for the transit authority's expectation of reducing the subway
system's operating deficit?
(A) Throughout the years that the subway system has operated, fares have never before been reduced.
(B) The planned fare reduction will not apply to students, who can already ride the subway for a reduced fare.
(C) Next year, the transit authority will have to undertake several large-scale track maintenance projects.
(D) The subway system can accommodate a ten percent increase in ridership without increasing the number of trains it runs
each day.
25. Some lovers speak when they their muses entertain,
Of hopes begot by fear, of wot not what desires,
Of force of heav’nly beams, infusing hellish pain,
Of living deaths, dear wounds, fair storms and freezing fires.
Some one his song in Jove, and Jove’s strange tales, attires,
Broidered with bulls and swans, powdered with golden rain.
Another, humbler, wit to shepherd’s pipe retires,
Yet hiding royal blood full oft in rural vein.
To some a sweetest plaint a sweetest style affords,
While tears pour out his ink, and sighs breathe out his words,
His paper, pale despair, and pain his pen doth move.
I can speak what I feel, and feel as much as they,
But think that all the map of my state I display,
When trembling voice brings forth, that I do Stella love.
Which choice best sums up the speaker’s main point?
(A) Others write long poems and use a lot of figurative language to express their feelings; I express just as much feeling when
I say simply “I love Stella.”
(B) I wish that I could express my love as others do, by writing beautiful and elaborate poems about Stella whom I love so
much.
(C) Since my muse is paying so much attention to other lovers, who spend all their time writing poems to their ladies, I can
only say “I love Stella.”
(D) I cannot compare my love for Stella to the love felt by poets who write beautiful poetry, because I can only say directly
that I love Stella.
26. Although the scientists were limited by the battery life of their instruments, they were able to collect data over the entire short
migration, as well as the first 140 days or so of the long migration. The GPS transmitters announced when the elephant seals

7
https://hocsat.com/
had returned to their home beaches. There, scientists used radio signals and plain old binoculars to pick out tagged seals from
the rest of the colony. After removing the loggers, they sent the seals back on their way.
What is the main effect of the phrase “plain old binoculars” ?
A) It suggests that the observations made with such binoculars may be unreliable.
B) It emphasizes the surprising usefulness of a simple observatory tool.
C) It reinforces that binoculars are obsolete compared to the radio signal used by the scientists
D) It establishes a definitive contrast between the goal of past and present researchers.
27. America, grandest of lands in the theory of its politics, in popular reading, in hospitality, breadth, animal beauty, cities, ships,
machines, money, credit, collapses quick as lightning at the repeated, admonishing, stern words, Where are any mental
expressions from you, beyond what you have copied or stolen? Where the born throngs of poets, literats, orators, you
promised? Will you but tag after other nations? They struggled long for their literature, painfully working their way, some with
deficient languages, some with priest-craft, some in the endeavor just to live—yet achieved for their times, works, poems,
perhaps the only solid consolation left to them through ages afterward of shame and decay.
According to the text, the author implies that American literature has so far borne what relationship to its English literary
inheritance?
A) It pays homage to English literature by alluding to it frequently.
B) It has to a large extent imitated English literature.
C) It has departed from the narrative conventions of English literature.
D) It has surpassed the quality of English literature.
28. And in a 1999 study of growth changes in tyrannosaurid skulls, paleontologist Thomas Carr found that none of the bone
fusions claimed by Gilmore or Bakker and colleagues were actually visible. This discovery, in addition to typical
characteristics of immature animals such as large, round orbits (eye sockets) and the texture of the bone, identified the skull as
a juvenile tyrannosaurid, most likely a young Tyrannosaurus rex. Given that tyrannosaurids were so _____________ and
underwent such dramatic changes from small, gracile juveniles into bulky, deep-skulled adults, it is little wonder that the
debate remains with us.
Which choice completes the text with the most logical and precise word or phrase?
A) adaptable.
B) diverse.
C) unspecified.
D) fickle.
29. Art restorers who have been studying the factors that cause Renaissance oil paintings to deteriorate physically when subject to
climatic changes have found that the oil paint used in these paintings actually adjusts to these changes well. The restorers
therefore hypothesize that it is a layer of material called gesso, which is under the paint, that causes the deterioration.
Which of the following, if true, most strongly supports the restorers' hypothesis?
A. Renaissance oil paintings with a thin layer of gesso are less likely to show deterioration in response to climatic changes
than those with a thickei layer.
B. Renaissance oil paintings are often painted on wooden panels, which swell when humidity increases and contract when it
declines.
C. Oil paint expands and contracts readily in response to changes in temperature, but it absorbs little water and so is little
affected by changes in humidity.
D. Gesso layers applied by Renaissance painters typically consisted of a coarse base layer onto which several increasingly fine
grained layers were applied
30. As animals age, the various bones that make up their skulls fiise aiorig sutures, and the degree to which the bones have fused
can sometimes be used to roughly determine age. Since all the skull bones in the Gilmour skull appeared to be fused, Bakker
and colleagues stated, the tyrannosaur must have been a small adult and therefore distinct from the bigger, bulkier

8
https://hocsat.com/
Tyrannosaurus rex. Appropriately, they called the hypothesized animal Nanotyrannus. Here's where things get tricky, though.
The timing of when sutures between skull bones fuse in dinosaurs varies among individuals and may not be a good indicator
of growth stage.
The method of determining a dinosaurs approximate age based on the degree of fusion of its skull bones produces results that
are
A) somewhat uncertain and subject to refutation.
B) more reliable than those of other available methods.
C) exact and indisputable.
D) typically of little value.
31. As Asch found, although the sway of group conformity is incredibly strong, it depends on unanimity for its power. In a
variation of the line study, Asch ran the experiment exactly as before (an unsuspecting participant, a room full of actors giving
the wrong answer), but this time he added a single actor who gave the right answer. This lone dissenting voice was enough to
break the spell, as it “gave permission” to the real participant to break ranks with the other members of the group.
Asch designed the study described in the text to investigate which question?
A) How might unanimity within a group influence the self—image of an individual in the group who privately disagrees with
the majority?
B) How might a lack of unanimity in a group influence an individual’s willingness to dissent openly from the majority?
C) How might an individual who dissents from the majority regain credibility within the group?
D) How might the intensity of an individual’s convictions impact the willingness of that individual to openly express them in a
group setting?
32. As the blubbery seals gained buoyancy, swimming became easier. They needed slightly more flipper strokes to make their
deep dives, but many fewer strokes to ascend. This meant that overall, fatter seals used fewer strokes to __________ the same
distance.
Which choice completes the text with the most logical and precise word or phrase?
A) spread out
B) shelter
C) deal with
D) travel
33. At one point, because Udayan suggested it, they began to linger outside Technicians’ Studio, where Satyajit Ray had shot
Pather Panchali, where Bengali cinema stars spent their days. Now and then, because someone who knew them was employed
on the shoot, they were ushered in amid the tangle of cables and wires, the glaring lights. After the call for silence, after the
board was clapped, they watched the director and his crew taking and retaking a single scene, perfecting a handful of lines. A
day’s work, devoted to a moment’s entertainment.
In describing the activity at the film studio, the narrator draws which contrast?
A) The calmness of the film shoot versus the fanfare that accompanies the debut of a film in movie theaters
B) The talent of the actors and film crew versus the quality of the finished film
C) The brother's interest in filmmaking versus the filmmaker's obliviousness to the boys
D) The time and effort that go into making a film versus the audience's fleeting experience of a film
34. At the beginning of each migration, the starved seals had “negative buoyancy.” In other words, they tended to sink. But as
their roving fish binge progressed, the seals became more and more buoyant. After two months at sea, all the seals were still
negatively buoyant, though their blubber had notably increased their buoyancy. After about five months, when the loggers
stopped gathering data for the long migration, 5 out of 7 seals had become “neutrally buoyant”—when drifting in the ocean,
they didn’t sink or rise. Adachi thinks the best state for elephant seals— the body type that keeps them swimming most
efficiently—is neutral buoyancy. Yet the hungry animals, gearing up for their next fast, keep eating beyond that.
Which finding, if accurate, would most clearly undermine Adachi’s belief that the northern elephant seals would keep gaining

9
https://hocsat.com/
blubber after the loggers stopped tracking the seals?
A) Evidence that the elephant seals do not stay negatively buoyant after the first month in the ocean.
B) Confirmation of the elephant seal’s tendency to consume increasingly greater quantities of food.
C) Proof that the elephant seals remain neutrally buoyant just before they return to the beaches to breed.
D) The discovery that the elephant seal reach their highest body mass before they return to the beaches to breed.
35. At the end of the experiment Dr. Johnson and his team collected the air inside the bags, extracted any volatile chemicals in it
by absorbing them into a special porous polymer, and tested those chemicals on both aphids and wasps. Each insect was
placed for five minutes in an apparatus that had two chambers, one of which contained a sample of the volatiles and the other
an odorless ______________.
Which choice completes the text with the most logical and precise word or phrase?
A) regulating force.
B) restrictive mechanism.
C) comparative element.
D) supervising factor.
36. At the Market Road Ranga dodged the traffic and paused in front of tailor’s and barber’s shops, offering his services. But those
were an erratic and unreliable lot, encouraging him by word but always suggesting another time for business. If they were not
busy cutting hair or clothes (tailors, particularly, never seemed to have a free moment, always stitching away on overdue
orders), they locked up and sneaked away, and Ranga had to be watchful and adopt all kinds of strategies in order to catch
them.
It can reasonably be inferred from the text that the tailors and barbers Ranga encounters are
A) strictly formal in their conduct.
B) somewhat hypocritical in their actions.
C) openly sarcastic in their responses.
D) unreasonably demanding in their expectations.
37. August was a hellish month to step off the train in Georgia, although it was nothing, Miss Grace Spivey said, compared to the
119 degrees that greeted her when she arrived one time in Timbuktu, which, she assured us, was a real place in Africa. I
believe her remark irritated some of the people gathered to welcome her on the burned grass alongside the tracks. When folks
are sweating through their shorts, they don’t like to hear that this is nothing compared to someplace else.
It can reasonably be inferred from the text that some of the people at the train station regard Miss Spivey's comment about the
Georgia heat with
A) sympathy, because they assume that she is experiencing intense heat for the first time.
B) disappointment, because they doubt that she will stay in Threestep for very long.
C) embarrassment, because they imagine that she is superior to them.
D) resentment, because they feel that she is minimizing their discomfort.
38. Because remote galaxies are so faint, the five target areas were away from our Milky Way’s star-studded plane. Much as
pollsters and medical researchers learn about the human population as a whole by studying carefully selected samples of a
small number of individuals, we chose the five target areas because they’re physically representative of the universe at large.
Based on the text, the authors would most likely say that conducting deep all-sky surveys to obtain information about ancient
galaxies is
A) practical.
B) unnecessary.
C) justified.
D) incomplete.
39. Before long people began talking in town about him. "Mir. Kohlah's screw is getting a little loose," they said. "He speaks to
trees and rocks, and pats them like they were his dogs."

10
https://hocsat.com/
When Maneck heard the gossip, he burned with shame, wishing his father would stop this embarrassing behaviour. He also
boiled with anger, wishing to slap some sense into the ignora insensitive people.
What does the text most strongly suggest about Maneck's attitude toward his father?
A) He feels conflicting emotions concerning his father.
B) He disapproves of his father's beliefs.
C) He is amused by his father's behavior.
D) He regards his father as backward and provincial.
40. Broad beans, then, really do seem to be using their fungal symbionts as a communications network, warning their neighbours
to take evasive action. Such a general response no doubt helps the plant first attacked by attracting yet more wasps to the area,
and it helps the fungal messengers by preserving their leguminous hosts.
Which choice best describes the nature of the relationship between the broad bean plants and fungi discussed in the text?
A) Mutually beneficial, since both organisms profit from the arrangement
B) Somewhat unbalanced, since one organism appears to benefit more than the other
C) Highly parasitic, since one organism benefits while the other experiences harm
D) Necessarily codependent, since neither organism can produce chemicals independently of the other
41. Budget constraints have made police officials considerreassigning a considerable number of officers from traffic enforcement
to work on higher-priority, serious crimes. Reducing traffic enforcement for this reason would be counterproductive, however,
in light of the tendency of criminals to use cars when engaged in the commission of serious crimes. An oftcer stopping a car
for a traffic violation can make a search ihat turns up evid0nce of serious crime.
Which of the following, if true, most strengthens the argument given?
(A) An officer who stops a car containing evidence of the commission of a serious crime risks a violent confrontation, even if
the vehicle was stopped only for a traffic violation.
(B) When the public becomes aware that traffic enforcement has lessened, it typically becomes lax in obeying traffic rules.
(C) Those willing to break the law to commit serious crimes are often in committing such crimes unwilling to observe what
they regard às the lesser constraints of traffic law.
(D) The offenders committing serious crimes who would be caught because of traffic violations are not the same group of
individuals as those who would be caught if the arresting officers were reassigned from traffic enforcement
42. Building good transit isn't a bad idea, but it can actually backfire if the new trains and buses merely clear space on highway
lanes for those who would prefer to drive a group that, historically, has included almost everyone with access to a car. To have
environmental value, new transit has to replace and eliminate driving on a scale sufficient to cut energy consumption overall.
That means that a new transit system has to be ____________ by something that impels complementary reductions in car use-
say, the physical elimination of traffic lanes or the conversion of existing roadways into bike or bus lanes, ideally in
combination with higher fuel taxes, parking fees, and tolls.
Which choice completes the text with the most logical and precise word or phrase?
A) supported.
B) copied.
C) substituted.
D) jammed.
43. Building good transit isn’t a bad idea, but it can actually backfire if the new trains and buses merely clear space on highway
lanes for those who would prefer to drive—a group that, historically, has included almost everyone with access to a car. To
have environmental value, new transit has to replace and eliminate driving on a scale sufficient to cut energy consumption
overall. That means that a new transit system has to be backed up by something that impels complementary reductions in car
use—say, the physical elimination of traffic lanes or the conversion of existing roadways into bike or bus lanes, ideally in
combination with higher fuel taxes, parking fees, and tolls. Needless to say, those ideas are not popular. But they’re necessary,
because you can’t make people drive less, in the long run, by taking steps that make driving more pleasant, economical, and

11
https://hocsat.com/
productive.
The author concedes that his recommendations are
A) costly to implement.
B) not widely supported.
C) strongly opposed by experts.
D) environmentally harmful in the short term.
44. But, when the researcher tested other corn varieties, individual results would vary. They repeated the experiment with different
___________ of corn from around the world. Like B73, some varieties supported more aphid offspring after a caterpillar
feeding, while the pre-feeding reduced the number of aphids or had no effect on other varieties.
Which choice completes the text with the most logical and precise word or phrase?
A) rows
B) fields
C) strains
D) routes
45. Caffeine started appearing in their droppings, and the animals themselves began, as it were, dropping off the perch. Over the
course of an experiment lasting 44 days after their guts had been sterilized (a period that let the insects complete an entire life
cycle of egg, larva, pupa and adult), the population of the experimental colonies fell by 95%—and even those larvae that did
not die had trouble pupating. Clearly, immunity to caffeine was being __________ by bacteria. The question was, which ones?
Which choice completes the text with the most logical and precise word or phrase?
A) donated
B) awarded
C) provided
D) served
46. CANDELS targeted five patches of the northern and southern skies, each about one-fourth the angular size of the Orion
Nebula (M42). Each ____________ has been well studied from radio to X rays, giving plenty of complementary data across
the electromagnetic spectrum.
Which choice completes the text with the most logical and precise word or phrase?
A) connection.
B) allotment.
C) covering.
D) region.
47. Captain Frederick Wentworth had come into Somersetshire in the summer of 1806; and having no parent living, found a home
for half a year, at Monkford. He was, at that time, a remarkably fine young man, with a great deal of intelligence, spirit and
brilliancy and Anne an extremely pretty girl, with gentleness, modesty, taste, and feeling. Half the sum of attraction, on either
side, might have been enough, for he had nothing to do, and she had hardly any body to love but the encounter of such lavish
recommendations could not fail. They were gradually acquainted, and when acquainted, rapidly and deeply in love. It would
be difficult to say which had seen highest perfection in the other, or which is had been the happiest; she, in receiving his
declarations and proposals, or he in having them accepted.
The underlined statement is best interpreted to mean that Anne and Captain Wentworth
A) were extremely generous to one another in all their interactions.
B) did not expect any opposition to their engagement.
C) were very confident of having success in life.
D) were so well matched that their attachment was inevitable.
48. Captain Wentworth had no fortune. He had been lucky in his profession, but spending freely what had come freely, had
__________ nothing. But, he was confident that he should soon be rich; full of life and ardour, he knew that he should soon

12
https://hocsat.com/
have a ship, and soon be on a station that would lead to everything he wanted. He had always been lucky; he knew he should
be so still.
Which choice completes the text with the most logical and precise word or phrase?
A) comprehended.
B) envisioned.
C) conceived.
D) gained.
49. Caro said regions with warmer, wetter climates are particularly susceptible to several species of disease- carrying flies other
than the tsetse flies that the team considered in their study, and that the relationship the researchers found may actually be a
function of fly avoidance, not thermoregulation.
the phrase “particularly susceptible” mainly serves to
A) point out a flaw in Larison’s conception of zebras’ geographic range.
B) emphasize the abundance of pests where some zebras live.
C) highlight the challenges the terrain presents to zebras in avoiding predators.
D) imply that the heat of their environment affects zebras more negatively than it does other grazers.
50. Carr argued that a comparison of Jane with the 1940s Nanotyrannus skull—made possible if both are considered juveniles—
kills the idea that the original Nanotyrannu skull has unique features that T.rex does not. "Jane was heralded by some as the
second coming of Nanotyrannus," Carr said. But according to his analysis, the two skulls share a number of features once
thought to be unique to Nanotyrannus, including a hole in a small jaw bone and a long and low snout. Rather than being
diagnostic of a separate species, Carr concluded, such features actually characterize juvenile tyrannosaurs.
Which factor, if true, would most undermine Carrs argument?
A) Identical skull structure evident in the Cleveland skull and the skull of ”Jane”
B) Presence of small holes in the jaw bones of other tyrannosaurs
C) Analysis identifying the Cleveland skull and Jane” as juvenile tyrannosaurs
D) Discovery of a small adult tyrannosaur other than Nanotyrannus with a long and low snout
51. Cats first entered human society about 9,500 years ago, not long after people first took up farming in the Middle East. Drawn
to rodents that had invaded grain stores, wildcats slunk out of the deserts and into villages. There, many scientists suspect, they
mostly domesticated themselves, with the friendliest ones able to take advantage of human table scraps and protection. Over
thousands of years, cats shrank slightly in size, acquired a panoply of coat colors and patterns, and (largely) shed the antisocial
tendencies of their past. Domestic animals from cows to dogs have undergone similar transformations, yet scientists know
relatively little about the genes involved.
The text suggests that some scientists would agree with which statement about the domestication of cats?
A) It did not occur until after the domestication of cows and pigs.
B) It caused cats to slightly increase in size.
C) It was not intentionally undertaken by humans.
D) It resulted from humans’ need for pets.
52. Charles Darwin was concerned with the collective and cumulative impact of earthworms in the wild.
On this count, he made large claims for them. He knew they were numerous, powerful, and busy. A German scientist had
recently come up with the figure 53,767 as the average earthworm population on each acre of the land he was studying, and to
Darwin this sounded about right for his own turf in England too. Every one of those 53,767 worms, he realized, spent much of
its time swallowing.
Which choice best describes the characterization of Darwin in the text?
A) The passage represents Darwin as a modern thinker whose ideas were criticized by the scientific community.
B) The passage represents Darwin as an insightful thinker who made expansive observations.

13
https://hocsat.com/
C) The passage depicts Darwin as an innovative thinker whose theories require only slight modifications.
D) The passage portrays Darwin as an imaginative thinker who tended to exaggerate the significance of his findings.
53. Charles Darwin was concerned with the collective and cumulative impact of earthworms in the wild. On this count, he made
large __________ them. He knew they were numerous, powerful, and busy. A German scientist had recently come up with the
figure 53,767 as the average earthworm population on each acre of the land he was studying, and to Darwin this sounded about
right for his own turf in England too. Every one of those 53,767 worms, he realized, spent much of its time swallowing.
Which choice completes the text with the most logical and precise word or phrase?
A) requests for
B) allowances for
C) assertions about
D) rights to
54. Cocoa grown organically on trees within the shade of the rain forest canopy commands a premium price. However, acquiring
and maintaining the certification that allows the crop to be sold as organically grown is very time-consuming and laborious.
Meanwhile, the price premium for the grower is about 30 percent, whereas cocoa trees grown in full sun using standard
techniques can have twice the yield of organic, /shade- grown trees. Financially, therefore, standard techniques are the better
choice for the farmer.
Which of the following, it true, most seriously weakens the arguments
A. Cocoa can be grown only in a climate that has the temperature and moisture characteristics of a tropical rain forest.
B. Cocoa trees grown using standard techniques require costly applications of fertilizer and pesticides, unlike shade-grown
trees.
C. Although organically grown cocoa has long commanded a price premium over cocoa grown using standard techniques, its
price has fluctuated considerably during that perlod.
D. Cocoa is not the only cash crop that can be raised on plots that leave the rain forest canopy overhead essentially intact.
55. Cooperation and parasitism could reciprocally influence one another: Cooperators might be more attractive targets because
they make better foster parents, but once exploited by the parasites, they are also better able to fight back. Feeney ct al. find
that superior anti-cuckoo defenses in larger groups account for more young fledged per season on average than smaller
groups—a substantial boost given the fairy-wrens‘ low annual fecundity
Which choice best describes reproduction in fairy-wrens, as discussed in the text?
A) Fairy-wrens choose lifelong males.
B) Fairy-wrens tend to produce few offspring.
C) Fairy-wrens lay a single clutch of eggs yearly.
D) Fairy-wrens are a capable of breeding year-round
56. Cultivation and refinement seem but to enhance their warmth of heart and ardent enthusiasm; and it is the possession of these
latter qualities in a most remarkable degree, which renders an educated American one of the most endearing and most
generous of friends. I never was so won upon, as by this class; never yielded up my full confidence and esteem so readily and
pleasurably, as to them; never can make again, in half a year, so many friends for whom I seem to entertain the _________ of
half a life.
Which choice completes the text with the most logical and precise word or phrase?
A) care.
B) motive.
C) appreciation.
D) aspect.
57. Data-based models suffer from their own shortcomings. In a paper published in 1995 Greg Mankiw of Harvard University
argued that they face insurmountable statistical problems. Too many things tend to happen at once to isolate cause and effect:
liberalised trade might boost growth, or liberalisation might be the sort of thing that governments do when growth is rising, or

14
https://hocsat.com/
both liberalisation and growth might follow from some third factor. And there are too many potential influences on growth for
economists to know whether a seemingly strong relationship between variables is real or would disappear if they factored in
some other relevant titbit, such as the wages of Canadian lumberjacks.
The text indicates that data-based forecasts are criticized on the grounds that they
A) cannot confidently determine how all economic variables interact.
B) include many economic variables that are likely to be of no importance.
C) assume that economic variable will be largely unchanged from year to year.
D) fail to consider economic variables that are included in theory-based forecasts.
58. If by dull rhymes our English must be chained,
And, like Andromeda, the Sonnet sweet
Fettered, in spite of pained loveliness;
Let us find out, if we must be constrained,
Sandals more interwoven and complete
To fit the naked foot of poesy;
Let us inspect the lyre, and weigh the stress
Of every chord, and see what may be gained
By ear industrous, and attention meet;
Misers of sound and syllable, no less
Than Midas of his coinage, let us be
Jealous of dead leaves in the bay wreath crown;
So, if we may not let the Muse be free,
She will be bound with garlands of her own.
Which choice best sums up the argument of the poem?
(A) We must find more truly poetic and meaningful ways to use rhyme in poetry.
(B) We must stop allowing the Muse to force us to use rhymes in poetry.
(C) We must do away with rhyme altogether if poetry is to survive as a form.
(D) We must recite our poems aloud to be sure of their quality before we publish.
59. Depending on the field, CANDELS took multiple images with exposure times ranging from 40 minutes to roughly 3 hours
through each of two or three infrared filters. Although CANDELS surveyed a total area only about that of the full Moon, the
long exposures looked so deep into the cosmos that they recorded roughly a quarter-million ancient galaxies in enough detail
to reveal their sizes, shapes, and even gross internal structures. Such a rich treasure trove provides powerful new data for
statistical studies of galaxy growth and evolution.
The reference to the full Moon primarily serves to
A) give readers a sense of the size of the area investigated by CANDELS.
B) identify the direction the HST was directed while collecting data for the HUDF.
C) help readers understand how images are processed by the Wide-Field Camera 3.
D) show how much farther away from Earth ancient galaxies are than the Moon is.
60. Disfluencies arise mainly because of the time pressures inherent in speaking. Speakers don't pre- plan an entire sentence and
then mentally press "play" to begin unspooling it. If they did, they'd probably need to pause for several seconds between each
sentence as they assembled it, and it's doubtful that they could hold a long, complex sentence in working memory. Instead,
speakers talk and think at the same time, launching into speech with only a vague ___________ of how the sentence will
unfold, taking it on faith that by the time they've finished uttering the earlier portions of the sentence, they'll have worked out
exactly what to say in the later portions.
Which choice completes the text with the most logical and precise word or phrase?
A)-experience.

15
https://hocsat.com/
B) touch.
C) significance.
D) idea.
61. Disfluencies arise mainly because of the time pressures inherent in speaking. Speakers don't pre-plan an entire sentence and
then mentally press "play" to begin unspooling it. If they did, they'd probably need to pause for several seconds between each
sentence as they assembled it, and it's doubtful that they could hold a long, complex sentence in working memory. Instead,
speakers talk and think at the same time, launching into speech with only a vague _____________ of how the sentence will
unfold, taking it on faith that by the time they've finished uttering the earlier portions of the sentence, they'll have worked out
exactly what to say in the later portions.
Which choice completes the text with the most logical and precise word or phrase?
A)-experience.
B) touch.
C) significance.
D) idea.
62. Disfluencies arise mainly because of the time pressures inherent in speaking. Speakers don’t pre- plan an entire sentence and
then mentally press “play‘ to begin unspooling it. If they did, they probably need to pause for several seconds between each
sentence as they assembled it, and it's doubtful that they could hold a long, complex sentence in working memory. Instead,
speakers talk and think at the same time, launching into speech with only a vague sense of how the sentence will unfold, taking
it on faith that by the time they've finished uttering the earlier portions of the sentence, they'll have worked out exactly what to
say in the later portions.
Which choice best paraphrases the authors point in the underlined portion?
A) Speakers do not allow suffcient time for selecting words appropriately.
B) Speakers do not construct complete sentences in their minds before uttering them.
C) Speaking is similar to mechanical forms of information retrieval.
D) Recording technologies unduly influence speech patterns.
63. Donations of imported food will be distributed to children in famine stricken countries in the form of free school meals. The
process is efficient because the children are easy to reach at the schools and cooking facilities are often available on site.
Which of the following, if true, casts the most serious doubt on the efficiency of the proposed process?
(A) The emphasis on food will detract from the major function of the schools, which is to educate the children. .
(B) A massive influx of donated food will tend to lower the price of food in the areas near the schools.
(C) Supplies of fuel needed for cooking at the schools arrive there only intermittently and in inadequate quantities.
(D) The reduction in farm surpluses in donor countries benefits the donor countries to a greater extent than the recipient
countries are bene5ted by the donations.
64. Dr. Johnson and his colleagues set up eight “mesocosms” [enclosed natural environments], each containing five beanstalks.
The plants were allowed to grow for four months, and during this time every plant could interact with symbiotic fungi in the
soil.
Not all of the beanstalks, though, had the same relationship with the fungi. In each mesocosm, one plant was surrounded by a
mesh penetrated by holes half a micron [0.0001 centimeter] across. Gaps that size are too small for either roots or hyphae to
penetrate, but they do permit the passage of water and dissolved chemicals. Two plants were surrounded with a 40-micron
mesh. This can be penetrated by hyphae but not by roots. The two remaining plants, one of which was at the centre of the
array, were left to grow unimpeded.
What is the main purpose of the text?
A) evaluate the experiment’s methods.
B) formulate the experiment’s goal.

16
https://hocsat.com/
C) document the experiment’s findings.
D) explain the experiment’s conditions.
65. Dr. Johnson knew from his own past work that when broad-bean plants are attacked by aphids they respond with volatile
chemicals that both irritate the parasites and attract aphid-hunting wasps. He did not know, though, whether the message could
spread from plant to plant. So he set out to find out—and to do so in a way which would show if fungi were the messengers.
The text suggests that in designing the experiment, Johnson relied on the fact that
A) fungal hyphae warn beanstalks of danger if a nearby plant is attacked.
B) wasps are harmful to the ongoing existence of broad bean plants.
C) broad bean plants release noxious chemicals to ward off infestation.
D) aphids are able to withstand the aggressive maneuvers used by wasps.
66. Earthworms were not only creating the planet’s thin layer of fertile soil; they were also constantly turning it inside out. They
were burying old Roman ruins. They were causing the monoliths of Stonehenge to subside and topple. On sloping land, where
rainwater and wind would sweep their castings away and down into valleys, they were making a huge contribution to erosion.
No wonder Darwin concluded: “Worms have played a more important part in the history of the world than most persons would
at first suppose.”
The repetition of the phrase “they were” serves primarily to
A) summarize a counterclaim to an argument presented earlier in the passage.
B) stress the likelihood that the hypotheses predicted by the author would occur.
C) offer competing interpretations of a fact presented in the passage.
D) emphasize the potential significance of an observation noted earlier in the paragraph.
67. Ecologists have shown that a farmer's field can have 50,000 or more weed seeds per square meter buried o beneath the soil
surface. Plant physiologists have shown that seeds buried more than about one centimeter below the soil surface do not receive
enough light to germinate. Do the blades of a plow, which can reach more than a foot beneath the soil surface, bring some of
these buried seeds to the surface where their germination is _________ by exposure to sunlight?
Which choice completes the text with the most logical and precise word or phrase?
A) lured.
B) established.
C) convinced.
D) stimulated.
68. Ecologists have shown that a farmer’s field can have 50,000 or more weed seeds per square meter buried beneath the soil
surface. Plant physiologists have shown that seeds buried more than about one centimeter below the soil surface do not receive
enough light to germinate. Do the blades of a plow, which can reach more than a foot beneath the soil surface, bring some of
these buried seeds to the surface where their germination is induced by exposure to sunlight?
According to the text, exposure to light allows seeds to
A) begin to develop.
B) absorb necessary nutrients.
C) withstand extreme temperatures.
D) achieve maximum growth.
69. Editorial: Consumers in North America think that by drinking frozen concentrated orange juice, they are saving energy,
because it takes fewer truckloads to transport it than it takes to transport an equivalent amount of not-from-concentrate juice.
But they are mistaken, for the amount of energy required to concentrate the juice is far greater than the energy difference in the
juices' transport.
Which of the following, if true, would provide the greatest additional support for the editorial's conclusions
(A) Freezer trucks use substantially more energy per mile driven than do any other types of trucks.
(B) Frozen juice can be stored for several years, while not-from-concentrate juice lasts a much shorter time.

17
https://hocsat.com/
(C) Oranges grown in Brazil make up an increasing percentage of the fruit used in not-from- concentrate juice production.
(D) A serving of not-from-concentrate juice takes up to six times more space than a serving of frozen concentrated juice.
70. Either food scarcity or excessive hunting can threaten a population of animals. If the group faces food scarcity, individuals in
the group will reach reproductive maturity later than otherwise. If the group faces excessive hunting, individuals that reach
reproductive maturity earlier will come to predominate. Therefore, it should be possible to determine whether prehistoric
mastodons became extinct because of food scarcity or human hunting, since there are fossilized mastodon remains from froth
before and after mastodon populations declined, and
Which of the following most logically completes the argument?
(A) there are more fossilized mastodon remains from the period before mastodon populations began to decline than from after
that period
(B) the average age at which mastodons from a given period reached reproductive maturity can be established from their
fossilized remains
(C) it can be accurately estimated from fossilized remains when mastodons became extinct
lD) it is riot known when humans first began hunting mastodons
71. Emma has sailed from Hong Kong to the United States in 1950 to attend college.
A sympathetic cabdriver, himself an immigrant from Russia, was kind and helpful. “You’re a smart one, taking taxi. These
docks is no good at night,” he said, his thick, bushy mustache moving up and down as he talked. “Don’t worry, Sergei is
getting you to your school nice and safe.”
Emma leaned forward and whispered to Sergei, “You are very kind.”
He quickly turned back with a smile, his warm breath touching her cheek. “I know how it feels.”
According to the text, Sergei treats Emma the way he does because
A) he understands what she is going through.
B) he enjoys showing off the sights of the city.
C) Emma reminds him of someone he knows.
D) Emma emigrated from his home country.
72. Emma has sailed from Hong Kong to the United States in 1950 to attend college.
A sympathetic cabdriver, himself an immigrant from Russia, was kind and helpful. “You’re a smart one, taking taxi. These
docks is no good at night,” he said, his thick, bushy mustache moving up and down as he talked. “Don’t worry, Sergei is
getting you to your school nice and safe.”
When Sergei first meets Emma, he assumes that she feels
A) intimidated by the college staff.
B) wary of accepting help.
C) daunted by her surroundings.
D) anxious about an unexpected expense
73. Emma has sailed from Hong Kong to the United States in 1950 to attend college.
Emma’s pulse raced. Chinatown appeared much smaller than she had expected. Restaurants and storefronts painted red, green,
and gold were crowded together into several blocks. She turned from window to window, soaking in all she could, seeking
echoes of the life she’d left behind.
Preoccupied faces she might have seen in Wanchai, or down in Causeway Bay, rushed down the bustling Grant Avenue.
Names of streets flashed by—Jackson, Pacific, and back to Broadway. As if he knew what she was thinking, Sergei circled
and drove through Chinatown again. Emma smiled, finding comfort in the Chinese characters written on signs and windows:
The Forbidden City Nightclub, Golden Harvest, Kuo Wah Restaurant, The Great Wall of China ... As different as this was
from Hong Kong, San Francisco’s Chinatown held the most familiar sights she’d seen in weeks.
Which choice best describes Emma’s overall perspective on Chinatown?
A) She is astonished that it has so many similarities with Hong Kong’s neighborhoods.

18
https://hocsat.com/
B) She is overwhelmed by the energetic pace of the people.
C) She is worried about initiating conversations with the local residents.
D) She is grateful to have found a place that reminds her of her hometown.
74. Every Englishman knows that they are a mere handful in this country and it is the _______________ of every one of them to
befool you in believing that you are weak and they are strong. This is politics. We have been deceived by such policy so long.
What the New Party wants you to do is to realize the fact that your future rests entirely in your own hands. If you mean to be
free, you can be free; If you do not mean to be free, you will fall and be for ever fallen.
Which choice completes the text with the most logical and precise word or phrase?
A) objective.
B) movement.
C) livelihood.
D) right.
75. Fatter seals can spend less energy swimming and more time eating, which gives them even more energy. So do they keep
gaining blubber indefinitely? “Yes, I think they get fatter to become positively buoyant,” Adachi says. If he could have
monitored the seals all the way to the end of their long migration, he thinks he would have seen them gain so much blubber
that they tended to float. Other research has found that elephant seals become positively buoyant, he adds.
It can most reasonably be inferred from the text that the overall conclusion reached by Adachi’s team might be challenged for
being
A) somewhat tenuous, because the team was unable to study the elephant seals throughout their entire migration.
B) ultimately inconclusive, because the team failed to expand the study to include a greater number of elephant seals from the
group.
C) moderately outdated, because the team used imprecise equipment to measure the buoyance of the elephant seals.
D) slightly unfocused, because the team incorporated extraneous information provided by other researches into their work.
76. Feeney ct al. find that superior anti-cuckoo defenses in larger groups account for more young fledged per season on average
than smaller groups—a substantial boost given the fairy-wrens‘ low annual fecundity.
These results show convincingly that defense against brood parasites augments the benefits of helping, promoting the
persistence of cooperation. But as the authors note, they cannot reveal what caused cooperation to evolve initially. Brood
parasitism alone cannot resolve the question of why some birds breed cooperatively. For example, cooperative kingfishers and
bee—eaters are heavily parasitized in Africa but not in Australia, showing that other advantages of helping behavior are
sufficient for cooperation to persist. But we should take parasitism seriously as an important force in a cooperative life.
The second paragraph primarily serves to
A) concede the difficulty of assessing the validity of the conclusions reached by Feeney et al.
B) reevaluate a basic premise of the study conducted by Feeney el al.
C) caution against an unjustified inference from the findings of Feeney et al.
D) note similarities between the data obtained by Feeney et al. and those of other studies.
77. Getting people to see the importance of keeping their edges sharp was indeed a tiresome mission. People’s reluctance and
lethargy had, initially, to be overcome. At first sight everyone dismissed him with “Go away, we have nothing to grind,” but if
he persisted and dallied, some member of the family was bound to produce a rusty knife, and others would follow, vying with
one another, presently, to ferret out long-forgotten junk and clamor for immediate attention. But it generally involved much
canvassing, coaxing and even aggressiveness on Ranga’s part.
The text suggests that Ranga’s methods of attracting business are
A) successful mainly as a result of determination on Ranga’s part.
B) ineffective, since few people value Ranga’s work.
C) comical because of Ranga’s eccentric appearance.
D) immediately fruitful because of Ranga’s unique skill.

19
https://hocsat.com/
78. GRAIL (Gravity Recovery and Interior Laboratory) is pair of NASA spacecraft that orbited the Moon in 2012. Sensitive to
tiny variations in the gravitational tug of the Moon, GRAIL mapped density variations below the surface (because regions of
higher density produces slightly higher gravitational forces). Below known impact basins, GRAIL found the expected ringlike
patterns, but underneath the Procellarum region, the mysterious rectangle emerged. “It was a striking pattern that demanded an
explanation,’ Andrews-Hanna says.
According to the text GRAIL was able to provide information about
A) gradations of shading in the large dark spot on the Moon’s near side.
B) fluctuations in the Moon's volcanic activity.
C) variations in the density of materials beneath the Moon’s surface.
D) forces influencing the gravitational tug of the Moon.
79. Håkan Olausson and his Gothenburg University colleagues Åke Vallbo and Johan Wessberg wondered if slow fibers
responsive to gentle pressure might be _________ in humans as well as in other mammals. In 1993, they corralled young
volunteers and recorded nerve signals while gently brushing the subjects' arms with their fingertips. Using a technique called
microneurography, in which a fine filament is inserted into a single nerve to capture its electrical impulses, the scientists were
able to measure how quickly-or slowly-the nerves fired.
Which choice complete the text with the most logical and precise word or phrase?
A) present.
B) attentive.
C) movable.
D) restless.
80. Håkan Olausson and his Gothenburg University colleagues Åke Vallbo and Johan Wessberg wondered if slow fibers
responsive to gentle pressure might be active in humans as well as in other mammals. In 1993, they corralled 28 young
volunteers and recorded nerve signals while gently brushing the subjects’ arms with their fingertips. Using a technique called
microneurography, in which a fine filament is inserted into a single nerve to capture its electrical impulses, the scientists were
able to measure how quickly—or slowly—the nerves fired. They showed that soft stroking prompted two different signals, one
immediate and one delayed.
Which conclusion is best supported by the findings of Olausson's 1993 experiment?
A) Stimulation at bodily extremities can be sensed as rapidly as stimulation closer to the brain.
B) The presence of hairs in human skin lessens the speed with which nerves conduct signals.
C) Gentle pressure is sensed not only by fast fibers but also by slow fibers.
D) The speed at which a nerve fires is dependent on the strength of pressure applied to the nerve.
81. Håkan Olausson and his Gothenburg University colleagues Åke Vallbo and Johan Wessberg wondered if slow fibers
responsive to gentle pressure might be active in humans as well as in other mammals. In 1993, they corralled young volunteers
and recorded nerve signals while gently brushing the subjects' arms with their fingertips. Using a technique called
microneurography, in which a fine filament is inserted into a single nerve to ___________ its electrical impulses, the scientists
were able to measure how quickly-or slowly-the nerves fired.
Which choice completes the text with the most logical and precise word or phrase?
A) occupy.
B) seize.
C) record.
D) influence.
82. Hartmann tested his idea by plowing two agricultural strips near Altershausen, Germany. The farmer Karl Seydel cultivated
one strip, repeated threefold, at around midday and the other strip at night. No crops were planted in these pilot experiments, to
avoid possible competition with the emerging weeds. The results were _________. More than 80 percent of the surface of the
field plowed in daylight was covered by weeds, whereas only about 2 percent of the field plowed at night was covered by

20
https://hocsat.com/
weeds.
Which choice completes the text with the most logical and precise word or phrase?
A) theatrical.
B) sudden.
C) impressive.
D) emotional.
83. Hattie looked closely at the crowd on the sidewalk. The Negroes did not step into the gutters to let the whites pass and they did
not stare doggedly at their own feet. Four Negro girls walked by, teenagers like Hattie, chatting to one another. Just girls in
conversation, giggling and easy, the way only white girls walked and talked in the city streets of Georgia. Hattie leaned
forward to watch their progress down the block.
The text makes which comparison regarding the four girls whom Hattie sees conversing with one another?
A) They appear as carefree in public as the white girls of Georgia did.
B) They dismiss Hattie a thoughtlessly as the white girls of Georgia did.
C) They discuss the same subjects that the white girls of Georgia discussed.
D) They lead home lives similar to those of the while girls of Georgia.
84. Hattie resisted the urge to cover her ears to block the rushing city sounds. She smelled the absence of trees before she saw it.
Things were bigger in Philadelphia---that was true—and there was more of everything, too mch of everything. But Hattie did
not see a promised land in this tumult. It was, she thought, only Atlantn on a larger scale. She could manage it. But even as she
declared herself adequate to the city, her knees knocked under her skirt and sweat rolled down her back.
Which choice best describes Hattie’s attitude upon first arriving in Philadelphia toward the prospect of living these?
A) She believes that Philadelphia has certain charms, but she worries that those charms are not genuine.
B) She knows that her identity will be altered by her new home, but she hopes hopes that this change will be for the better.
C) She recognizes that metropolitan areas offer many advantages, but she would still prefer to return to the country.
D) She feels that she is capable of residing in such a large city, but the possibility of doing so unnerves her.
85. Hattle was only fifteen then, slim as a finger. She stood with her mother and sisters at the crowd'k edge, the four of them
waiting for a break in the flow of people so they too might move toward the double doors at the for end of the atatlom Hattie
stepped into the muhitudc. Mama called, ‘Come back! You'll be lost in all those people. You’ll be lost!‘ Hattie looked back in
panic; ahe thought her mother was right behind her. The crowd was too thick for her to turn back. and she was borne along on
the current of people. She ___________ the double doors and was pushed out onto a long sidewalk that ran the length of the
station.
Which choice completes the text with the most logical and precise word or phrase?
A) reached.
B) increased.
C) attracted.
D) defeated.
86. He must have told me hundreds, perhaps even thousands, of stories, not just at night, but throughout the course of any given
day, over breakfast, during lunch, in the middle of a conversation he might have been carrying on with my mother or friends.
There was no wrong time with him, or if there was, he didn’t live long enough for me to see it.
The text indicates that the narrator’s father viewed storytelling as something that
A) continued a long-standing family tradition.
B) provided a means of bringing historical events to life.
C) helped children to distinguish good choices from bad ones.
D) was part of the fabric of the family’s everyday life.
87. How does the brain ___________ when assessing these two different types of scenarios? The results were intriguing. Common
to both types of situation was some level of mental activity in parts of the brain, such as the hippocampus, that are at work

21
https://hocsat.com/
when we in general recall facts or events. Such activity was detectable regardless of the nature of the scenario—that is,
whether the scenario was informative (when Peter only heard about the characters) or interactive (when he actually met the
characters).
Which choice completes the text with the most logical and precise word or phrase?
A) manipulate.
B) exercise.
C) function.
D) conduct.
88. I had more customers then, and I treated each interruption to our reading as an assault on my privacy. When someone I didn’t
know entered the store, Naomi would mark where I had left off so that I could keep my eyes on the person wandering around
the aisles. She would take the book out of my hand, put her finger on the exact word or sentence I had just ______________,
and hold it there until I returned. I kept one man, who came to the counter with a single roll of toilet paper under his arm,
waiting for more than a minute while I finished reading a page I had just started.
Which choice completes the text with the most logical and precise word or phrase?
A) decided.
B) finished.
C) inferred.
D) dismissed.
89. I hold that Banks’s policy is our chief danger at the present moment; that it practically enslaves the Negro, and makes the
Emancipation Proclamation of 1863 a mockery and delusion. What is freedom? It is the right to choose one’s own
employment. Certainly it means that, if it means anything; and when any individual or combination of individuals undertakes
to decide for any man when he shall work, where he shall work, at what he shall work, and for what he shall work, he or they
_____________ reduce him to slavery.
Which choice completes the text with the most logical and precise word or phrase?
A) effectively.
B) reasonably.
C) cleverly.
D) partially.
90. I looked up every couple of pages to see if Naomi was still paying attention, and of course she was. Her attention, in fact,
never seemed to waver. I felt her staring at me sometimes when my eyes were focused on the page, and I realized she was
taking it all in, not just the words, but me, and the scene that we had created together.
The narrator’s descriptions of Naomi suggest that she is notable for her
A) empathy toward other people.
B) ability to concentrate and observe.
C) talent for dramatizing fiction.
D) optimism despite many hardships
91. I see a government anxiously bent on the public good. Even in its errors I recognize a paternal feeling towards the great people
committed to its ____________ I see the public mind of India... expanding itself to just and noble views of the ends of
government and of the social duties of man.
Which choice completes the text with the most logical and precise word or phrase?
A) care.
B) invasion.
C) accusation.
D) expense.

22
https://hocsat.com/
92. I slipped into the characters as I read. I grumbled and bellowed, slammed my fist onto the counter, and threw my arms wide
open. I knew this was exactly what my father would have done had he been the one reading. He would have made the story an
event, as grand and real as life.
It can reasonably be inferred from the text that the narrator’s father influenced him by
A) creating an environment that encouraged young people to express themselves.
B) emphasizing what was most important in a life well lived.
C) providing a model for the dramatic recounting of a story.
D) exposing the narrator to a number of classic novels written by renowned authors.
93. I slipped into the characters as I read. I grumbled and bellowed, slammed my fist onto the counter, and threw my arms wide
open. I knew this was exactly what my father would have done had he been the one reading. He would have made the story an
event, as grand and real as life. He must have told me hundreds, perhaps even thousands, of stories, not just at night, but
throughout the course of any given day, over breakfast, during lunch, in the middle of a conversation he might have been
carrying on with my mother or friends. There was no __________ time with him, or if there was, he didn’t live long enough
for me to see it.
Which choice completes the text with the most logical and precise word or phrase?
A) inappropriate.
B) unjust.
C) immoral.
D) inaccurate
94. If I did bring myself to sell all my pictures and sketches, they would not give me twenty kopeks for the whole of them. They
are useful; I feel that not one of them has been undertaken in vain; I have learned something from each one. Yes, but of what
use is it? Studies, sketches, all will be studies, trial-sketches to the end. And who will buy, not even knowing me by name?
Who wants drawings from the antique, or the life class, or my unfinished love of a Psyche, or the interior of my room, or the
portrait of Nikita, though it is better, to tell the truth, than the portraits by any of the fashionable artists? Why do I worry, and
toil like a learner over the alphabet, when I might shine as brightly as the rest, and have money, too, like them?
The text suggests that to some extent, the narrator finds maintaining his high artistic standards to be a
A) means of attaining short-lived fame as opposed to a lasting reputation.
B) goal less important for his professor than it is for himself.
C) necessary pathway to a goal he now seeks to accomplish.
D) laborious undertaking that does not provide suitable compensation.
95. If I had heard the story before, I let him tell it to me again. His performance was that good, his love of a story that obvious.
When Fyodor Karamazov spoke, I waved my hands wildly in the air. I grumbled in a deep baritone and tried as hard as I could
to do my father proud.
Which situation is most similar to the one described in underlined lines?
A) A viewer eagerly anticipates the new episodes of a television series each week.
B) An artist paints a favorite landscape at several different hours of the day
C) A moviegoer happily sees a particularly memorable film for a second time.
D) An actress repeatedly recites the lines of her script to improve her performance.
96. If you cut commuting time by 10%, people who now drive fifty miles each way to work can justify moving five miles farther
out, because their travel time won’t change. This is how metropolitan areas metastasize. It’s the history of suburban expansion.
According to the text, reducing commuting time for drivers can have which of the following effects?
A) Drivers become more productive employees than they previously were.
B) Mass transit gets extended farther into suburban areas than it previously was.
C) Mass transit carries fewer passengers and receives less government funding than it previously did.
D) Drivers become more willing to live farther from their places of employment than they previously were.

23
https://hocsat.com/
97. If, in the distant future, archaeologists find no ___________ of evidence of our civilization apart from a library of 20th century
novels, they might be able to figure out something surprising about recent history: the boom times and recessions of our
economic system.
Which choice completes the text with the most logical and precise word or phrase?
A) pattern.
B) path.
C) sketch.
D) remnant.
98. In 1999, the group looked more closely at the characteristics of the slow fibers. They named these “low-threshold” nerves “C-
tactile,” or CT fibers, said Olausson, because of their “exquisite sensitivity” to slow, gentle tactile stimulation, but
unresponsiveness to noxious stimuli like pinpricks.
But why exactly humans might have such fibers, which respond only to a narrow range of rather subtle stimuli, was initially
mystifying. Unlike other types of sensory nerves, CT fibers could be found only in hairy human skin—such as the forearm and
thigh. No amount of gentle stroking of hairless skin, such as the palms and soles of the feet, prompted similar activity
signatures. Olausson and his colleagues decided that these fibers must be conveying a different dimension of sensory
information than fast-conducting fibers.
The underlined portion serves mainly to
A) identify factors that Olausson had previously failed to consider.
B) propose a solution to a dilemma encountered by Olausson.
C) anticipate a potential criticism of Olausson by the reader.
D) show a problem from the perspective of Olausson's team.
99. In 2001, another skull and partial skeleton similar to that of the claimed Nanotyrannus were found in Montana. Nicknamed
“Jane," it was better preserved than the earlier 1940s specimen, and many researchers concluded it was a juvenile based on the
shape of its teeth and other skeletal features. Indeed, a few former Nanotyrannus advocates changed their minds based on the
new skull. But others continued to argue that it was a separate species.
Compared with the specimen found by Gilmore in the 1940s, the remains of the dinosaur nicknamed Jane found in 2001 are
A) in a more complete state.
B) form a different geological era.
C) more difficult to classify.
D) From a different continent
100. In a variation of the line study, Asch ran the experiment exactly as before (an unsuspecting participant, a room full of actors
giving the wrong answer), but this time he added a single actor who gave the right answer. This lone dissenting voice was
enough to break the spell, as it “gave permission” to the real participant to break ranks with the other members of the group.

24
https://hocsat.com/
The figure helps support a main claim advanced in the text by showing that the
A) presence of a dissenter decisively shifted the opinion of the majority.
B) presence of a dissenter improved the accuracy of responses only when there were multiple dissenters.
C) accuracy of responses given by participants when a dissenter was present varied across trials.
D) accuracy of responses given by participants was higher overall when a dissenter was present.
101. In an experiment, paired birds of matching head colour were moved into an experimental cage without food. After one hour of
food deprivation a feeder was placed into the corner of the cage where there was only enough room for one bird to feed at a
time. Aggressive interactions such as threat displays and displacements were then counted over a 30-minute period. The results
were striking. Red-headed birds were significantly and consistently more aggressive than black-headed birds
The author uses the word “displacements” most likely to suggest that one bird
A) replaces the other in a scale of social dominance.
B) intimidates the other so that it flees the cage.
C) fights the other until one signals submission.
D) jostles the other aside to access the food supply
102. In another set of experiments, undergraduate students were given a calendar on a Wednesday and were asked to suggest an
appropriate day to carry out certain tasks before the following Sunday. The trick was that some were given a calendar with all
of the weekdays coloured purple, with weekends in beige (making a visual distinction between a Wednesday and the following
Sunday). Others were given a calendar in which every other week, Monday to Sunday, was a solid colour (meaning that a
Wednesday and the following Sunday were thus in the same week, and in the same colour). Even this minor visual cue
affected how like— or unlike—the—present the respondents tended to view task priorities.
These and other bits of framing and trickery in the research support the same thesis: that making people link a future event to
today triggers an implemental response, regardless of how far in the future the deadline actually lies.
The primary purpose of the second paragraph is to
A) propose the need for further study of a claim.
B) emphasize the strength of a research conclusion.

25
https://hocsat.com/
C) summarize the methods used in an experiment.
D) recommend a solution to a research challenge
103. In Asch’s study each participant was placed in a room with several other people. The participants were told they would be
tested for visual acuity. The task seemed simple enough: the group was shown three straight lines of varying lengths, and each
person was asked to determine which of the three lines matched a fourth line. It was pretty straightforward; the lengths were so
glaringly different that you certainly didn’t need a magnifying glass or a ruler. But what the participant didn’t know was that
the other “subjects” in the room were really actors, and all of them had been instructed to give the same wrong answer. As the
actors called out their erroneous answers one by one, the real participant was bewildered. But something strange happened:
rather than stick to their guns, most participants began to doubt themselves and their lone dissenting opinion.
The authors put the word “subjects” in quotation marks most likely to
A) emphasize the strategy employed to mislead the study participants.
B) suggest the extent to which the experiment’s outcome was predictable.
C) underscore the impartial nature of Asch’s selection of participants for the experiment.
D) convey the capability of the actors who took part in Asch’s experiment.
104. In her mind were running scenes of the play. Particularly she remembered one beautiful actress—the sweetheart who had been
wooed and won. The grace of this woman had won Carrie’s heart. Her dresses had been all that art could suggest, her
sufferings had been so real. The anguish which she had portrayed Carrie could feel. It was done as she was sure she could do
it. There were places in which she could even do better. Hence she repeated the lines to herself. Oh, if she could only have
such a part, how broad would be her life! She, too, could act appealingly.
According to the text, Carrie judges the performance of the actress who played the hero’s sweetheart as
A) inferior in certain respects to the performance that she believes herself to be capable of giving.
B) dissatisfying in its emphasis on physical grace over emotional authenticity.
C) clumsy and melodramatic yet superior to the other actors’ performances.
D) difficult to watch because of the intense suffering it evoked.
105. In its search for new technologies for solar energy that can produce lower-cost electricity, Smith-Diatom is developing a new
way to make dye-sensitive solar cells, in which photons strike light-sensitive dyes. The process uses diatoms, which are
unicellular algae that have silicon shells with a complex structure. First, the organic material is removed, and then the shells
are coated with a titanium dioxide film that acts as a semiconductor. The diatoms’ structure results in more photon activity and
thus more eftcient production of electricity than with current dye-sensitized solar cells, which in turn lowers the cost.
Which of the following considerations would, if true, most strongly support the hypothesis that the plan, if implemented, will
produce low-cost electricity from dye-sensitive solar cells?
(A) Diatoms are an important link in oceanic food chains and help cycle carbon dioxide from the atmosphere.
(B) The current cost of electricity produced by dye- sensitive solar cells is roughly three times that of non-solar electricity
(C) Because diatoms occur naturally, no special engineering processes are needed to produce the basic dye-sensitive solar cell
structures.
(D) Dye-sensitive solar cells work somewhat more efficiently in lower light than previous solar cell technologies.
106. In parts of South America, vitamin-A deficiency is a serious health problem, especially among children. In one region,
agriculturists are attempting to improve nutrition by encouragihg farmers to plant a new variety of sweet potato called SPK004
that is rich in beta- carotene, which the body converts into vitamin A. The plan has good chances of success, since sweet
potato is a staple of the region's diet and agriculture, and the varieties currently grown contain little beta-carotene.
Which of the following, if true, most strongly supports the prediction that the plan will succeed?
(A) The growing conditions required by the varieties of sweet potato currently cultivated in the region are conditions in which
SPK004 can flourish.
(B) The flesh of SPK004 differs from that of the currently cultivated sweet potatoes in color and texture, so traditional foods
would look

26
https://hocsat.com/
somewhat different when prepared from SPK004.
There are no other varieties of sweet potato that are significantly richer in beta-carotene than SPK004 is.
(D) The varieties of sweet potato currently cultivated in the region contain some important nutrients that are lacking in
SPK004.
107. In practice, most forecasters combine the two approaches and inject, when necessary, a dose of common sense. The IMF, for
instance, relies on a global model, built in part on economic theory and in part on data analysis. The global projections
generated by that hybrid model are combined with country-specific details to __________ country-level forecasts. The country
forecasts are then checked for consistency against the global projections and adjusted when necessary-to make sure, for
example, that most countries do not show strong trade growth when the global projection heralds a decline in trade.
Which choice completes the text with the most logical and precise word or phrase?
A) create
B) exhibit
C) cause
D) extend
108. In spite of their differences one was perpetually confused with the other, so that when either name was called both were
conditioned to answer. And sometimes it was difficult to know who had answered, given that their voices were nearly
indistinguishable. Sitting over the chessboard they were mirror images: one leg bent, the other splayed out, chins propped on
their knees.
They were similar enough in build to draw from a single pile of clothes. Their complexions, a light coppery compound derived
from their parents, were identical. Their double-jointed fingers, the sharp cut of their features, the wavy texture of their hair.
In the context of the narrator’s physical description of the brothers, the detail of the “single pile of clothes” serves mainly to
A) Emphasize that the description mirrors the brothers’ perception of themselves.
B) Note one respect in which the description may misrepresent the brothers.
C) Cite a particular fact about the brothers that corroborates the description.
D) Suggest that the description holds truer are certain times than at others.
109. In the early 1990s, textbooks acknowledged that humans had slow-conducting nerves, but asserted that those nerves only
responded to two types of stimuli: pain and temperature. Sensations of pressure and vibration were believed to travel only
along myelinated, fast-signaling nerve fibers, which also give information about location. Experiments blocking nerve fibers
supported this notion. Preventing fast fibers from firing (either by clamping the relevant nerve or by injecting the local
anesthetic lidocaine) seemed to eliminate the sensation of pressure altogether, but blocking slow fibers only seemed to reduce
sensitivity to warmth or a small painful shock.
Based on the text, textbook authors in the early 1990s would most likely have expected which condition to result from the
blocking of fast fibers?
A) The rate at which other nerve fibers fired would increase.
B) The test subject would perceive gentle stimuli as painful.
C) The body would compensate by using slow fibers sense pressure.
D) The ability to perceive vibrations would be impaired.
110. In the first eighty thousand years of their one- hundred—thousand-year window, Nevada lake sticklebacks had almost no
protective weapons. But then, eighty—four thousand years into the time sequence, this type of stickleback was replaced
entirely by armored sticklebacks, meaning three long dorsal spines and full pelvic spines. Bell suspects that marine fish
flooded into the lake around this time, because both forms co- occurred for about one hundred years before the early fish type
disappeared. Remarkably, over the following thirteen thousand years, the defensive structures in this new fish regressed: in
graded steps through time, the spines got shorter and shorter, until by the end of this period the new sticklebacks resembled the
earlier form that theyil replaced. Like-bound fish lost their weapons.
The statement “Lake-bound fish lost their weapons” mainly serves to

27
https://hocsat.com/
A) criticize a study discussed in the paragraph.
B) provide an example in support of a claim made m the paragraph.
C) summarize a process described in detail in the paragraph.
D) emphasize the importance of a study discussed paragraph.
111. In the hawkmoth Manduca sexta, the LPO is large and apparently not sexually dimorphic, suggesting that in this species, CO2,
information could be similarly important for both males and females. We speculated, therefore, that information about ambient
CO₂, could be valuable for functions other than, or in addition to, oviposition. Manduca species, which feed as adults, possess
a more __________ LPO than that of moths that do not feed as adults. We hypothesize, therefore, that Manduca might use its
CO₂-sensing system to detect the high metabolic activities of flowers and thus to locate profitable nectar sources.
Which choice completes the text with the most logical and precise word or phrase?
A) ornamented.
B) developed.
C) meticulous.
D) showy.
112. Is it enough that we have emancipation and abolition upon the statute books? In some states of society, I should say yes. In
ancient times when the slaves were of the same race with their masters, when the slaves were poets, orators, scholars, ministers
of state, merchants, and the mothers of kings—if they were emancipated, nature came to their aid, and they reached an equality
with their masters. Their children became patricians. But, my friends, this is a slavery of race; it is a slavery which those white
people have been taught, for thirty years, is a divine institution. I ask you, has the Southern heart been _______________ for
thirty years for nothing? Have those doctrines been sown, and no fruit reaped? Have they been taught that the negro is not fit
for freedom, have they believed that, and are they converted in a day? Besides all that, they look upon the negro as the cause of
their defeat and humiliation. . . .
Which choice completes the text with the most logical and precise word or phrase?
A) dismissed.
B) illuminated.
C) propelled.
D) roused.
113. It is an essential part of every national character to pique itself mightily upon its faults, and to deduce tokens of its virtue or its
wisdom from their very exaggeration. One great blemish in the popular mind of America, and the prolific parent of an
innumerable brood of evils, is Universal Distrust. Yet the American citizen plumes himself upon this spirit, even when he is
sufficiently dispassionate to perceive the ruin it works; and will often adduce it, in spite of his own ____________, as an
instance of the great sagacity and acuteness of the people, and their superior shrewdness and independence.
Which choice completes the text with the most logical and precise word or phrase?
A) judgment.
B) explanation.
C) cause.
D) defense.
114. It is our voice that sang "America" when America grew too lazy, satisfied, and confident to sing, before the dark threats and
fire-lined clouds of destruction frightened it into a __________, panicky quaver. We are more than a few isolated instances of
courage, valor, achievement. We're the injection, the shot in the arm, that has kept America and its gotten principles alive in
the fat and corrupt years intervening between our divine conception and our near-tragic present.
Which choice completes the text with the most logical and precise word or phrase?
A) slender.
B) weak.

28
https://hocsat.com/
C) sparse.
D) fine.
115. It is to be seen that advertising is not an economic waste. It ministers to the true development of trade. It is no doubt possible
to waste money through wrong methods of advertising, as it can be wasted through wrong methods in any department of
industry. But rightfully applied, it is the method by which the desire is created for better things. When that once exists, new
ambition is developed for the creation and use of wealth. The uncivilized make little progress because they have few desires.
The inhabitants of our country are stimulated to new wants in all directions. In order to satisfy their constantly increasing
desires they necessarily expand their productive power. They create more wealth because it is only by that method that they
can satisfy their wants. It is this constantly enlarging circle that represents the increasing progress of civilization.
According to the text, advertising is useful to society because it
A) helps people understand the overarching goals of the government.
B) energizes the economy by encouraging people to want new products.
C) inspires a sense of shared cultural identity among people.
D) allows people to uncover the deceptions of some manufacturers.
116. It is true that the duties of government and legislation were long wholly neglected or carelessly performed [by the British in
India]. It is true that when the conquerors at length began to apply themselves in earnest to the discharge of their high
functions, they committed the errors natural to rulers who were but imperfectly acquainted with the language and manners of
their subjects. It is true that some plans, which were dictated by the purest and most benevolent feelings, have not been
__________ by the desired success.
Which choice completes the text with the most logical and precise word or phrase?
A) frequented.
B) maintained.
C) replaced.
D) accompanied.
117. It seemed to him that the nineteenth century had improved upon them considerably, that the delineation of nature was more
clear, more vivid, more close. It sometimes vexed him when he saw how a strange artist, French or German, sometimes not
even a painter by profession, but only a skilful dauber, produced, by the celerity of his brush and the vividness of his
colouring, a universal commotion, and amassed in a twinkling a funded capital. This did not occur to him when fully occupied
with his own work, for then he forgot food and drink and all the world. But when dire ____________ arrived, when he had no
money wherewith to buy brushes and colours, when his implacable landlord came ten times a day to demand the rent for his
rooms, then did the luck of the wealthy artists recur to his hungry imagination; then did the thought which so often traverses
Russian minds, to give up altogether, and go down hill, utterly to the bad, traverse his. And now he was almost in this frame of
mind.
Which choice completes the text with the most logical and precise word or phrase?
A) need.
B) absence.
C) ambition.
D) greed.
118. It was in Barnard College, she told us, in the midst of trying to find her true mission in life, that she wandered one afternoon
into a lecture by the famous John Dewey, who was talking about his famous book, Democracy and Education. Professor
Dewey was in his seventies by then, Miss Spivey said, but he still liked to chat with students after a lecture—especially female
students, she added—sometimes over coffee, and see in their eyes the fire his words could kindle. It was after this lecture and
subsequent coffee that Miss Spivey had marched to the Teacher’s College and signed up, all aflame. Two years later, she told
a cheery blue-suited woman from the WPA that she wanted to bring democracy and education to the poorest, darkest, most
remote and forgotten corner of America.

29
https://hocsat.com/
They sent her to Threestep, Georgia.
What is the narrator most likely suggesting by describing Miss Spivey as having "wandered" in one situation and "marched" in
another situation?
A) Dewey, knowing Miss Spivey wasn't very confident in her ability to teach, instilled in her a sense of determination.
B) Talking with Dewey over coffee made Miss Spivey realize how excited she was to teach in the poorest, most remote corner
of America.
C) After two years spent studying, Miss Spivey was anxious to start teaching and be in charge of her own classroom.
D) Miss Spivey's initial encounter with Dewey's ideas was somewhat accidental but ultimately motivated her to decisive
action.
119. When most I wink, then do mine eyes best see,
For all the day they view things unrespected;
But when I sleep, in dreams they look on thee,
And darkly bright are bright in dark directed.
Then thou, whose shadow shadows doth make bright,
How would thy shadow’s form form happy show
To the clear day with thy much clearer light,
When to unseeing eyes thy shade shines so!
How would, I say, mine eyes be blessed made
By looking on thee in the living day,
When in dead night thy fair imperfect shade
Through heavy sleep on sightless eyes doth stay!
All days are nights to see till I see thee,
And nights bright days when dreams do show thee me.
Which of the following can you conclude about the relationship between the speaker and the beloved?
I. They are in love with each other and happy together.
II. They were in love long ago but no longer see one another.
III. They have not seen one another for some time.
(A) I only
(B) I and II only
(C) II only
(D) III only
120. It was not often that she came to the play _____________ to her heart’s core by actualities. To-day a low song of
longing had been set singing in her heart by the finery, the merriment, the beauty she had seen.
Which choice completes the text with the most logical and precise word or phrase?
A) moved.
B) angered.
C) prodded.
D) encouraged.
121. JEROME FRANK, a mid-20th-century legal thinker, is said to have claimed that justice is a __________ of what the judge
had for breakfast. Don't let their black robes, serious miens and pledges of fealty to the law fool you, Mr Frank warned:
judicial decisions are not cool applications of objective legal principles. Rather, they are manifestations of personal
predilections and biases.
Which choice completes the text with the most logical and precise word or phrase?
A) result.
B) occupation.

30
https://hocsat.com/
C) celebration.
D) purpose.
122. Jerome Frank, a mid-20th-century legal thinker, is said to have claimed that justice is a function of what the judge had for
breakfast. Don’t let their black robes, serious miens and pledges of fealty to the law fool you, Mr. Frank warned: judicial
decisions are not cool applications of objective legal principles. Rather, they are manifestations of personal predilections and
biases.
Indeed, new research by three political scientists shows just how avidly the justices go to bat for causes they identify with. In
their paper, Lee Epstein of the University of Southern California and two colleagues examined 4.519 votes in 516 Supreme
Court free-speech cases from 1953 to 2010 to determine whether "justices defend the speech they hate." The answer: rarely.
How does Maize use the discussion of Frank and Epstein to present his argument?
A) He cites Frank as an established authority for a recommended course of action and Epstein as a more contemporary
authority.
B) He cites Frank in order to endorse a theory and Epstein in order to suggest a slight modification to that theory.
C) He cites Frank as a spokesperson for a particular position and Epstein as a researcher corroborating that position.
D) He cites Frank in order to identify a long-standing dilemma and Epstein as a
scientist offering a solution to that dilemma.
123. Just how do we know that the accumulation of Ice Age glaciers lowered sea level by 120 meters? Well, it would be possible to
calculate the amount of ocean surface lowering if you knew exactly how much ice was on the land, but that too is a difficult
question to answer. You would need to estimate how thick the ice was, and what area of the land it covered. What seems a
simple problem suddenly looks quite complicated, and to solve it required a great deal of ingenuity.
Which choice best states the structure of the text?
A) describe the difficulties in adapting a common method of data analysis to new circumstances.
B) acknowledge a possible objection to an argument and conclude that the objection is baseless.
C) show how a seemingly complicated question was broken down into several simpler questions.
D) indicate a potential approach to a problem and explain why that approach is not feasible.
124. Kill P. fulva, then, and you would probably kill the borer. But that is easier said than done. Even if spraying coffee plantations
with antibiotics were feasible and would do the job (by no means certain, for the larvae would have to ingest sufficient
antibiotic for the purpose), it would be undesirable. The profligate use of antibiotics encourages resistance, thus making them
less effective for saving human lives.
The underlined portion primarily serves to
A) acknowledge a potential impediment to implementing a proposal supported by the author.
B) imply that current limitations on adopting an approach the author favors may soon be overcome.
C) call into question the intentions of researchers who have suggested a plan that the author opposes.
D) emphasize the impracticality of a course of action that the author describes as harmful regardless.
125. Larison found that the plains zebras with the most- defined torso stripes generally lived in the Northern, equatorial region of
their range, whereas those with less-defined torso stripes were more common in the Southern, cooler regions of the range—a
finding that supports the thermoregulation explanation.
Still, the researchers have not experimentally tested the theory that black and white stripes may generate small- scale breezes
over a zebra’s body, and some researchers don’t think stripes can actually create this effect.
The text indicates that Larison’s team’s study falls short of being definitive because Larison
A) disregarded facts that did not support her conclusions.
B) used research methods that have not proved effective.
C) did not build on the achievements of prior research on the subject.
D) has yet to confirm a key assumption made in the study.

31
https://hocsat.com/
126. Leaf beetles damage willow trees by stripping away their leaves, but a combination of parasites and predators generally keeps
populations of these beetles in check. Researchers have found that severe air pollution results in reduced predator populations.
The parasites, by contrast, are not adversely affected by pollution; nevertheless, the researchers' discovery probably does
explain why leaf beetles cause particularly severe damage to willows in areas with severe air pollution, since ____________
Which of the following most logically completes the argument below?
A. neither the predators nor the parasites of leaf beetles themselves attack willow trees
B. the parasites that attach leaf beetles actually tend to be more prevalent in areas with severe air pollution than they are
elsewhere
C. the damage caused by leaf beetles is usually not enough to kill a willow tree outright
D. where air pollution is not especially severe, predators have much more impact on leaf-beetle populations than parasites do
127. Leaves, twigs and other plant debris had accumulated on the forest floor over the years and created a thick layer of what is
called duff. Fungi, bacteria and microscopic invertebrates such as mites slowly broke down this debris. The duff held moisture
like a sponge, nurturing the growth of many understory plants like wildflowers, shrubs and tree seedlings. Small animals and
birds nested and fed on the forest floor and in understory foliage.
The author of the text conceives of the duff as
A) the foundation of the forest ecosystem.
B) a common entry point for invasive species.
C) the local manifestation of global environmental change.
D) an ongoing safeguard against further habitat loss.
128. Look here, my friend,” his professor said to Tchartkoff more than once. "At present your colouring begins to assert itself too
loudly; and your drawing is at times quite weak; you are already striving after the fashionable style, because it strikes the eye
at once. Have a care! Society already begins to have its attraction for you: I have seen you with a shiny hat, a foppish
neckerchief...It is seductive to paint fashionable little pictures and portraits for money; but talent is ruined, not developed, by
that means. Be patient; think out every piece of work, discard your foppishness; let others amass money, your own will not fail
you.”
The text suggests that Tchartkoff’s professor believes that great art should be
A) technically accomplished and not garish.
B) pleasing to the eye but not overly popular.
C) original in approach and spontaneous in execution.
D) representative of the artist’s morals and beliefs.
129. Manufacturers are now required to make all cigarette lighters child-resistant by equipping them with safety levers. But this
change is unlikely to result in a significant reduction in the number of fires caused by children playing with lighters, because
children given the opportunity can figure out how to work the safety levers and
Which of the following, if true, most logically complete the argument?
(A) the addition of the safety levers has made lighters more expensive than they were before the requirement was instituted
(B) adults are more likely to leave child-resistant lighters than non-child-resistant lighters in places that are accessible to
children
(C) many of the fires started by young children are quickly detected and extinguished by their parents
(D) unlike child-resistant lighters, lighters that are not child-resistant can be operated by children as young as two years old
130. Many scientists think that our cultural fixation with stamping out what they call "disfluencies" is deeply misguided. Saying um
is no character flaw, but an organic ___________ of speech; far from distracting listeners, there's evidence that it focuses their
attention in ways that enhance comprehension.
Which choice completes the text with the most logical and precise word or phrase?
A) purpose.
B) report.

32
https://hocsat.com/
C) component.
D) appearance.
131. Marine reserves, areas that are closed to all fishing, have been attracting much attention for their dual line potential as
conservation and fishery management tools. A synthesis of more than 100 studies of 15 reserves worldwide shows that
protection from fishing leads to rapid increases in biomass, abundance, and average size of exploited organisms and to
increased species diversity. Such effects are of great interest to fishery managers because rebuilding exploited populations in
reserves offers prospects of fishery enhancement.
According to the text, observable changes to fish living in marine reserves include increases in
A) total population and size of individuals.
B) the viability and hardiness of fish eggs.
C) life expectancies and growth rates of larger species.
D) the number of fish migrating into the reserves.
132. Market forecaster: The price of pecans is high when pecans are comparatively scarce but drops sharply when pecans are
abundant. Thus, in high-yield years, growers often store part of their crop in refrigerated warehouses until after the next year's
harvest, hoping for higher prices then. Because of bad weather, this year's pecan crop will be very small. Nevertheless, pecan
prices this year will not be significantly higher than last year, since
Which of the following most logically completes the argument?
(A) the last time the pecan crop was as small as it was this year, the practice of holding back part of one year's crop had not yet
become widely established
(B) last year's pecan harvest was the largest in the last 40 years
(C) pecan prices have remained relatively stable in recent years
(D) pecan yields for some farmers were as high this year as they had been last year
133. Miss Grace Spivey was, by almost anyone’s standards, a woman of the world. She’d gone to boarding schools since she was
six years old; she’d studied French in Paris and drama in London; and during what she called a “fruitful intermission” in her
formal education, she had traveled extensively in the Near East and Africa with a friend of her grandmother’s, one Janet
Miller, who was a medical doctor from Nashville, Tennessee.

Miss Spivey uses the phrase "fruitful intermission" to indicate that


A) she benefited from taking time off from her studies in order to travel.
B) her travels with Janet Miller encouraged her to start medical school.
C) her early years at boarding school resulted in unanticipated rewards.
D) what she thought would be a short break from school lasted several years.
134. Miss Spivey showed us on the map how she and Dr. Janet Miller had sailed across the Atlantic Ocean and past the Rock of
Gibraltar into the Mediterranean Sea. Using the end of a ruler, she gently tapped such places as Morocco and Tunis and
Algiers to mark their route along the top of Africa. They spent twenty hours on the train to Baghdad, she said, swathed in veils
against the sand that crept in every crack and crevice.
“And can you guess what we saw from the train?” Miss Spivey asked. We could not. “Camels!” she said. “We saw a whole
caravan of camels.” She looked around the room, waiting for us to be amazed and delighted at the thought.
We all hung there for a minute, thinking hard, until Mavis Davis spoke up.
In the text, when Miss Spivey announces that she had seen camels, the students' reaction suggests that they are
A) delighted.
B) fascinated.
C) baffled.
D) worried.

33
https://hocsat.com/
135. More recently, Karl Hartmann of Erlangen University in Germany reasoned that when farmers plowed their fields during the
day, the buried weed seeds are briefly exposed to sunlight as the soil is turned over, and that this stimulates their germination.
Hartmann tested his idea by plowing two agricultural strips near Altershausen, Germany. The farmer Karl Seydel cultivated
one strip, repeated threefold, at around midday and the other strip at night. No crops were planted in these pilot experiments, to
avoid possible competition with the emerging weeds. The results were dramatic. More than 80 percent of the surface of the
field plowed in daylight was covered by weeds, whereas only about 2 percent of the field plowed at night was covered by
weeds.
The text suggests that if Seydel had planted wheat or corn on the two agricultural strips in Hartmann's experiment, the
percentage of the surface of each strip covered with weeds would likely have been
A) lower than the percentage that Hartmann found.
B) higher than the percentage that Hartmann had predicted.
C) nearly impossible for Hartmann to determine.
D) comparable to Hartmann's original projection.
136. “Mabel Osborne” is a 1915 poem by Edgar Lee Masters in which thenarrator expresses her sorrow at being ignored by other
people.
Which quotation from “Mabel Osborne” most effectively illustrates that claim?
A. “But you do not ask for water. / You cannot speak!”
B. “Everyone knows that you are dying of thirst, / Yet they do not bringwater!”
C. “And I, who had happiness to share, / And longed to share your happiness”
D. “I who loved you, Spoon River, / And craved your love,”
137. Most banks that issue credit cards charge interest rates on credit card debt that are ten percentage points higher than the rates
those banks charge for ordinary consumer loans. These banhs' representatives claim the difference is fully justified, since it
simply covers the difference between the costs to these banks associated with credit card debt and those associated with
consumer loans.
Which of the following, if true, most seriously calls into question the reasoning offered by the banks' representativesf
A. Some lenders that are not banks offer consumer loans at interest rates that are even higher than most banks charge on credit
card debt
B. Most car rental companies require that their customers provlde signed credit card charge slips or security deposits.
C. Two to three percent of the selling price of every item bought with a given credit card goes to the bank that issued that
credit card.
D. Most people need not use credit cards to buy everyday necessities, but could buy those necessities with cash or pay by
check.
138. Mr. Frank's observation seems to _______________ to today's Supreme Court. When ruling on big, controversial cases, the
justices split fairly reliably along party lines dictated by their appointing presidents. It wasn't always this way. Until 1937,
party simply wasn't a factor in high-court decisions.
Which choice completes the text with the most logical and precise word or phrase?
A) plead successfully.
B) relate especially.
C) appeal deeply.
D) stick physically.
139. Ms. Tu and Dr. Soman advise in their paper that “the key step in getting things done is to get started'’ But what ________ that?
They believe the key that unlocks the implemental mode lies in how people categorise time. They suggest that tasks are more
likely to be viewed with an implemental mindset if an imposed deadline is cognitively linked to “now”— a so-called like-the-
present scenario.
Which choice completes the text with the most logical and precise word or phrase?

34
https://hocsat.com/
A) prompts.
B) constraint.
C) accelerates.
D) persuades.
140. Ms. Tu and Dr. Soman carried out five sets of tests, with volunteers ranging from farmers in India to undergraduate students in
Toronto. In one test, the farmers were offered a financial incentive to open a bank account and make a deposit within six
months. The researchers predicted those approached in June would consider a deadline before December 31st as like—the—
present. Those approached in July, by contrast, received a deadline into the next year, and were expected to think of their
deadline as unlike—the-present. The distinction worked. Those with a deadline in the same year were nearly four times more
likely to open the account immediately as those for whom the deadline lay in the following year.

Which choice best states an important difference between the results of the test described in the text and the data presented in
the graph?
A) The study in the passage concluded that subjects are unwilling to start tasks considered unlike- the-present, whereas the
graph shows that such tasks are eventually completed.
B) The study in the passage analyzed what subjects actually did, whereas the graph reports on what subjects said that they
would do.
C) The study in the passage was conducted over a relatively short period of time, whereas the graph reflects data collected
over several months.
The study in the passage found only a small difference between like-the-present and unlike- the-present scenarios, whereas the
difference shown by the data in the graph is more substantial.
141. Naomi, an eleven-year-old on a school break, is visiting the narrator, a family friend from Ethiopia, at the store he manages.
They have made a plan to read the novel The Brothers Karamazov together.
I tried not to notice too much, to simply just live, but that was impossible. Every time I looked at her I became aware of just
how seemingly perfect this reading time was. I thought about how years from now I would remember this with a crushing,
heartbreaking nostalgia, because of course I knew even then that I would eventually find myself standing here alone.
Which choice best reflects the perspective of the narrator regarding his reading to Naomi?
A) He is unequivocally delighted, because reading to Naomi has provided him with a much-needed creative outlet.
B) He is pleasantly surprised, because he did not expect Naomi to be so enthusiastic about reading.

35
https://hocsat.com/
C) He is occasionally regretful, because he is not as skilled a reader as his father was.
D) He is somewhat ambivalent, because he knows that his reading time with Naomi will inevitably end.
142. Nearly all black holes fall into one of two classes: big, and colossal. But scattered across the universe like oases in a desert are
a few apparent black holes of a more mysterious type. Ranging from a hundred times to a few hundred thousand times the
sun’s mass, these intermediate-mass black holes are so hard to measure that even their existence is sometimes disputed. Little
is known about how they form. And some astronomers question whether they behave like other black holes.
The text indicates that research into intermediate-mass black holes has been impeded by the fact that
A) such black holes seem to have a comparatively short life span.
B) current models of the universe’s evolution cannot explain such black holes.
C) it has been difficult for scientists to precisely measure such black holes.
D) the gravitational pull that such black holes exert on other objects is negligible.
143. New York must be filled with such bowers, or the beautiful, insolent, supercilious creatures could not be. Some hothouses held
them. It ached her to know that she was not one of them—that, alas, she had dreamed a dream and it had not come true. She
wondered at her own solitude these two years past—her indifference to the fact that she had never achieved what she had
expected.
Based on the text, Carrie regards her day-to-day lifestyle as one that has
A) not been conducive to helping her achieve her ambitions.
B) followed a repetitive but productive pattern.
C) allowed for interaction with a range of interesting people.
D) facilitated her goals as an actress.
144. No matter how much we know about the world of a fictional character there will still be something _________________ and
inscrutable to us about that world. You may have read all the books about a fictional character, but the amount of information
you have gathered about that character is still definitely limited compared with the wealth of information that is available to
you about members of your family, friends, or famous real people who are part of your immediate and past experience.
Basically, in order to understand a fictional character, you need to dig deeper into your imagination, because he or she is
bound up to fewer nodes of reference in your network than are real, or relevant, people in your life.
Which choice completes the text with the most logical and precise word or phrase?
A) inconsistent.
B) foreign.
C) extraterrestrial.
D) complex.
145. No specific type of bacteria seemed more important than another in ____________ the vaccine response. But further
experiments showed a major role for macrophages—immune cells that display pieces of the virus to activate B cells and that
can also recognize flagellin. Pulendran’s favored explanation is that flagellin manages to break through the lining of the
intestines to circulate in the body and activate B cells and macrophages, amping up antibody production. But where and how
the interaction happens “is a huge mystery,” he says. “We don’t have the full answer.”
Which choice completes the text with the most logical and precise word or phrase?
A) inciting.
B) suggesting.
C) shortening.
D) refreshing.
146. No specific type of bacteria seemed more important than another in prompting the vaccine response. But further experiments
showed a major role for macrophages—immune cells that display pieces of the virus to activate B cells and that can also
recognize flagellin. Pulendran’s favored explanation is that flagellin manages to break through the lining of the intestines to
circulate in the body and activate B cells and macrophages, amping up antibody production. But where and how the interaction

36
https://hocsat.com/
happens “is a huge mystery,” he says. “We don’t have the full answer.”
What is the main purpose of the text?
A) question the overall importance of the vaccine response in light of new discoveries.
B) suggest that the researchers who studied the vaccine response were uncertain about their data.
C) emphasize the fact that further research into the nature of the vaccine response is necessary.
D) urge other scientists to consider furthering the existing work regarding the vaccine response.
147. Not surprisingly, this variation in weapon size influences fish survival. Long spines make sticklebacks difficult to swallow,
and armor plates protect sticklebacks whenever predatory fish make the mistake of trying to bite them. Almost 90 percent of
attacks on sticklebacks fail. But before spitting them out, predators chew sticklebacks rather harshly. A stickleback's armor
plates act like shields, reducing the ___________ of injuries from these bites.
Which choice completes the text with the most logical and precise word or phrase?
A) duration.
B) degree.
C) length.
D) area.
148. Nutritionists are advising people to eat more fish, since the omega 3 fatty acids in fish help combat many diseases. If everyone
took this advice, however, there would not be enough fish in oceans, rivers, and lakes to supply the demand; the oceans are
already being overfished. The obvious method to ease the pressure on wild fish populations is for people to increase their
consumption of farmed fish.
Which of the following, if true, raises the most serious doubt concerning the prospects for success of the solution proposed
above?
(A) Aquaculture, or fish farming, raises more fish in a given volume of water than are generally present in the wild.
(B) Some fish farming, particularly of shrimp and other shellfish, takes place in enclosures in the ocean.
(C) There are large expanses of ocean waters that do not contain enough nutrients to support substantial fish populations.
(D) The feed for farmed ocean fish is largely made from small wild-caught fish, including the young of many popular food
species.
149. Occasionally Ranga would warn, “If you do not sharpen your articles now, you may not have another chance, since I am going
away on a pilgrimage.”
“Makes no difference, we will call in the other fellow,” someone would say, referring to a competitor, a miserable fellow who
operated a hand grinder, collected his cash and disappeared, never giving a second look to his handiwork. He was a fellow
without a social standing, and no one knew his name, no spark ever came out of his wheel, while Ranga created a regular
pyrotechnic display and passing children stood transfixed by the spectacle.
The text suggests that compared to his competitor, Ranga is perceived by certain people as being
A) less sentimental in his beliefs.
B) less sophisticated in his technology.
C) more interested in his profits.
D) more impressive in his presentation.
150. Of course, this correlation, whether in the U.S., Great Britain or Germany, might not come as a huge shock-obviously, the
circumstances that surround an author influence his or her word choices. But the fact that the signal of economic times could
be consistently spotted through the _________ of all of an author's personal circumstances is still somewhat surprising, and
shows what a profound effect economics have on our creative mindsets.
Which choice completes the text with the most logical and precise word or phrase?
A) irrelevant information.
B) static interference.

37
https://hocsat.com/
C) common gossip.
D) positive attention.
151. Often patients with ankle fractures that are stable, and thus do not require surgery, are given follow-up x-rays because their
orthopedists are concerned about possibly having misjudged the stability of the fracture. When a number of follow-up x-rays
were reviewed, however, all the fractures that had initially been judged stable were found to have healed correctly. Therefore,
it is a waste of money to order follow-up x-rays of ankle fractures initially judged stable.
Which of the following, if true, most strengthens the arguments
(A) Doctors who are general practitioners rather than orthopedists are less likely than orthopedists to judge the stability of an
ankle fracture correctly.
(B) Many ankle injuries for which an initial x-ray is ordered are revealed by the x-ray not to involve any fracture of the ankle.
(C) X-rays of patients of many different orthopedists working in several hospitals were reviewed.
(D) The healing of ankle fractures that have been surgically repaired is always checked by means of a follow-up x ray.
152. Olausson’s group sought out a patient known as G.L. who had an unusual nerve defect. More than 2 decades earlier, she had
developed numbness across many parts of her body after taking penicillin to treat a cough and fever. Olausson used functional
MRI studies to examine which areas of the brain lit up when G.L.’s arm was gently brushed to activate CT fibers. In normal
subjects, both the somatosensory and insular cortices were activated, but only the insular cortex, which processes emotion, was
active when researchers brushed G.L.’s arm.
According to the passage, G.L.. differed from Olausson's other test subjects in terms of the
A) number of cortices activated in the brain during gentle brushing.
B) physical dimensions of the somatosensory cortex.
C) intensity of nerve signals required to activate the insular cortex.
D) effect of MRI scanning on the basic function of brain cortices.
153. In Infiite City. A San Francisco Atlas, a 2010 collection by Rebecca Solnit, each of the atlas’s twenty-two maps tells a distinct
story. A map pinpointing the city’s coffee shops also charts sewer lines and reservoirs to show how commerce, infrastructure,
and natural resources are interrelated. Another relays the rise and decline of cinema culture in San Francisco by juxtaposing
sites where Alfred Hitchcock filmed his iconic movie Vertigo with locations where movie theaters have recently closed. Of
this collection, L.A. Times reviewer Lynell George wrote,
Which quotation from “Lynell George” most effectively illustrates the point made in the collection by Rebecca Solnit?
A. “We often speak of inhabiting a place—a country, a city or our own small plot of land— but seldom do we pause to
deeply consider how that place inhabits us.”
B. “Infinite City examines that San Francisco, a physically compact place that contains multitudes.”
C. “This book alters the focus and point of view, it tells the story of a city through the voices of its inhabitants
obsessions, dreams, predilections, passions. It allows what lies beneath the surface to speak.”
D. “Passing through these newly mapped territories, we begin to see that ‘place,’ as Solnit emphasizes, is an imprecise
word.”
154. One hundred and twenty meters over the entire ocean adds up to a very large amount of water, about 3 percent of the present
ocean volume. All of this water, evaporated from the sea, was transported to the continents as water vapor in the atmosphere,
fell as snow, and accumulated as the glaciers of the Ice Age. Where the ice was thick, as in parts of Canada and Scandinavia,
for example, an enormous weight was placed on a relatively small area of the continental crust. Slowly but steadily, the crust
actually sank down into the yielding rocks of the underlying mantle in response to this burden. When the ice melted, these
same areas began to rebound, and they have been slowly rising for the past ten thousand years or more.
It can reasonably be inferred from the text that in parts of Canada and Scandinavia there are
A) forces beneath the continental crust pushing the crust upward.
B) signs that the crust has sunk since the end of the last ice age.

38
https://hocsat.com/
C) greater accumulations of water vapor than in other areas at similar latitudes.
D) many mantle rocks present on the surface
155. One of the arguments that cities inevitably make in promoting transit plans is that the new system, by relieving automobile
congestion, will improve the lives of those who continue to drive. No one ever ___________ a transit scheme by arguing that
it would make traveling less convenient-even though, from an environmental perspective, inconvenient travel is a worthy goal.
Which choice complete the text with the most logical and precise word or phrase?
A) upgrades.
B) serves.
C) advocates.
D) develops.
156. One of the few forces with a proven ability to slow the growth of suburban sprawl has been the ultimately finite tolerance of
commuters for long, annoying commutes. That means that traffic congestion can have environmental value, since it lengthens
commuting times and, by doing so, discourages the proliferation of still more energy-hungry subdivisions— unless we made
the congestion go away. If, in a mis- guided effort to do something of environmental value, municipalities take steps that make
long-distance car commuting faster or more convenient—by adding lanes, building bypasses, employing traffic-control
measures that make it possible for existing roads to accommodate more cars with fewer delays, replacing tollbooths with
radio-based systems that don’t require drivers even to slow down—we actually make the sprawl problem worse, by indirectly
encouraging people to live still farther from their jobs, stores, schools, and doctors’ offices, and by forcing municipalities to
further extend road networks, power grids, water lines, and other civic infrastructure.
Based on the text, how would the author most likely characterize many attempts to improve traffic?
A) They are doomed to fail because most people like driving too much to change their habits.
B) They overestimate how tolerant people are of long commutes.
C) They are well intentioned but ultimately lead to environmental harm.
D) They will only work if they make driving more economical and productive.
157. One of the most important effects of the Pleistocene Ice Age is something that is difficult to observe visually but is quite
__________ if you think about it a bit. It is the fact that there was a massive transfer of water from the oceans to the land
during the glacial periods. It has been estimated that at the maximum of the most recent glacial period, about twenty thousand
years ago, sea level was approximately 120 meters lower than it is at present. Along most shorelines, dry land extended far out
into what is now quite deep water.
Which choice completes the text with the most logical and precise word or phrase?
A) commonplace.
B) reliable.
C) unsurprising.
D) well known.
158. One of the most important effects of the Pleistocene Ice Age is something that is difficult to observe visually but is quite
__________ if you think about it a bit. It is the fact that there was a massive transfer of water from the oceans to the land
during the glacial periods. It has been estimated that at the maximum of the most recent glacial period, about twenty thousand
years ago, sea level was approximately 120 meters lower than it is at present. Along most shorelines, dry land extended far out
into what is now quite deep water.
Which choice completes the text with the most logical and precise word or phrase?
A) commonplace.
B) reliable.
C) unsurprising.
D) well known.

39
https://hocsat.com/
159. One of the most important effects of the Pleistocene Ice Age is something that is difficult to observe visually but is quite
predictable if you think about it a bit. It is the fact that there was a massive transfer of water from the oceans to the land during
the glacial periods. It has been estimated that at the maximum of the most recent glacial period, about twenty thousand years
ago, sea level was approximately 120 meters lower than it is at present. Along most shorelines, dry land extended far out into
what is now quite deep water. A map of the world as it was then would look quite different from the one we are familiar with
today.
The text suggests that a map of the world as it was 20,000 years ago would differ from a map of the world as it is today in
which way?
A) Mountain chains would be closer to shorelines.
B) Rivers would be longer but less numerous.
C) The ratio of land to sea would be greater.
D) Islands would appear farther apart.
160. Other animals also need to regulate body temperature, or thermoregulate, Larison pointed out, but zebras may especially
benefit from an extra cooling system because they digest food much less efficiently than other grazers in Africa. As such,
zebras need to spend longer periods of time out in the heat of the midday sun, eating more food.
The reason stripes are particularly beneficial to zebras is probably because zebras
A) endure greater exposure to the midday sun than other grazers do.
B) cannot escape predators as easily as other grazers in hot regions can.
C) live in hot, dry climates that lack an adequate food supply for most grazers.
D) defend themselves more poorly against predator attacks than other grazers do.
161. Our findings indicate that in 5 years, reserves have led to improvement in the Soufriere Management Marine Area fishery,
despite the 35% decrease in area of fishing grounds. There were more fish in the sea, and evidence for little 70 initial impact of
reserves on total catches in the first year of implementation, together with constant fishing effort since protection began,
indicates a greater weight of total landings. Interviews with local fishers (conducted in Creole via an interpreter) showed that
most felt better off with reserves than without. Younger fishers were especially positive about the benefits.
The authors include the information about the local fishers primarily in order to
A) demonstrate the beneficial impacts of the study on residents' lives.
B) suggest that the analysis of the study data was argely subjective.
C) lend anecdotal support to findings the researchers gathered through scientific observation.
D) present an alternative explanation for the results reported for the period between 2000 and 2001.
162. Our streets are in mourning, tears are falling at every fireside, and under the chastisement of this Rebellion we have almost
come up to the point of conceding this great, this all-important right of suffrage. I fear that if we fail to do it now, we may not
see, for centuries to come, the same ____________ that exists at this moment.
Which choice completes the text with the most logical and precise word or phrase?
A) habit.
B) placement.
C) settlement.
D) attitude.
163. Pasham focused on one object in Messier 82, a galaxy in the constellation Ursa Major. Messier 82 is our closest “starburst
galaxy,” where young stars are forming. Beginning in 1999 a NASA satellite telescope, the Chandra X-ray Observatory,
detected X-rays in Messier 82 from a bright object prosaically dubbed M82 X-1. Astronomers, including Mushotzky and co-
author Tod Strohmayer of NASA’s Goddard Space Flight Center, suspected
for about a decade that the object was an intermediate-mass black hole, but estimates of its mass were not definitive enough to
confirm that.
It can reasonably be inferred from the passage that the data collected by the Chandra X-ray Observatory were insufficient to

40
https://hocsat.com/
rule out which possibility?
A) M82 X-1 is a member of one of the two established classes of black holes.
B) The X-rays associated with M82 X-1 come from another object entirely.
C) Changes in the brightness of M82 X-1 reflect imperfections in the telescope itself.
D) M82 X-1 was formed later than was any known supermassive black hole.
164. PhishCo runs a number of farms in the arid province of Nufa, depending largely on irrigation. Now, as part of a plan to
efficiently increase the farms' total production, it plans to drill down to an aquifer containing warm, slightly salty water that
will be used to raise fish in ponds. The water from the ponds will later be used to supplement piped-in irrigation water for
PhishCo's vegetable fields, and the ponds and accompanying vegetation should help reduce the heat in the area of the farms.
Which of the following would, if true, most strongly suggest that the plan, if implemented, would increase the overall
efficiency of PhishCo's farms?
(A) Most of the vegetation to be placed around the ponds is native to Nufa.
(B) Fish raised on PhishCo's farms are likely to be saleable in the nearest urban areas.
(C) Organic waste from fish in the pond water will help to fertilize fields where it is used for irrigation.
(D) The government of Nufa will help to arrange loan financing to partially cover the costs of drilling.
165. Place a housecat next to its direct ancestor, the Near Eastern wildcat, and it may take you a minute to spot the difference.
They’re about the same size and shape, and, well, they both look like cats. But the wildcat is fierce and feral, whereas the
housecat, thanks to nearly 10,000 years of domestication, is tame and adaptable enough to have become the world’s most
popular pet. Now scientists have begun to pinpoint the genetic changes that drove this remarkable transformation. The
findings, based on the first high-quality sequence of the cat genome, could shed light on how other creatures become tame.
The author includes the underlined phrase most likely to
A) stress that the relationship between wildcats and domestic cats is continually shifting.
B) introduce a discussion of the physical attributes of domestic cats.
C) advance a tentative hypothesis about the evolution and development of wildcats.
D) acknowledge the obviousness of a similarity between wildcats and domestic cats.
166. Ranga would retort, “I do not grudge the poor fellow his luck, but he will impart to your knife the sharpness of an egg; after
that I won’t be able to do anything for you. You must not think that anyone and everyone could handle steel. Most of these
fellows don’t know the difference between a knife blade and a hammerhead.”
Ranga refers to an egg in order to emphasize which aspect of the knife?
A) Its fragility
B) Its bluntness
C) Its ordinariness
D) Its utility
167. Ranga’s customers loved his banter and appreciated his work, which he always guaranteed for sixty days. “If it gets dull before
then, you may call me.” If he were to be assailed for defective execution, he could always turn around and retort that so much
depended upon the quality of metal, and the action of sun and rain, and above all the care in handling, but he never argued with
his customers; he just resharpened the knives free of cost on his next round. Customers always liked to feel that they had won
a point, and Ranga would say to himself, “After all, it costs nothing, only a few more turns of the wheel and a couple of sparks
off the stone to please the eye.”
The information about Ranga’s sixty—day work guarantee serves mainly to
A) emphasize Ranga’s astute knowledge of how to conduct business.
B) show Ranga’s cleverness in outsmarting his customers.
C) describe Ranga’s attempt to match his competitor’s strategy.
D) introduce a reason for Ranga’s looming financial failure.

41
https://hocsat.com/
168. Red-headed birds should be bolder, more explorative and take more risks than black-headed birds. This hypothesis is based on
previous studies of other species that have shown a correlation between aggression and these behavioural characteristics.
However, there is another possibility. Red-headed birds could take fewer risks for two reasons: first, they may be more
conspicuous to predators due to their bright colouration and second, it may pay black-headed birds to take more risks and be
more explorative so they find food resources before the dominant red-headed birds do.
It can reasonably be inferred that the prediction reflects which assumption?
A) Risk taking is more beneficial to black-headed finches than to red-headed ones.
B) Aggressive behaviors in black- and red-headed finches will be comparable.
C) Observed correlations between certain behaviors in one species translate to other species.
D) Innate and acquired behaviors in birds are often difficult to distinguish.
169. Red-headed birds were considerably more cautious than those with black heads at returning to the feeder after a “predator” had
been introduced. They took on average four times longer to begin feeding again than the less aggressive black-headed birds.
Red-headed birds were more aggressive than black-headed finches but took fewer risks and were not explorative.
Red colouration has been found to be conspicuous against natural backgrounds, and more conspicuous birds have been found
to suffer higher predation rates. Thus, selection could favour more conspicuous red-headed birds taking fewer risks.
Interestingly, boldness [in investigating novel objects] and risk-taking behaviours were found to be strongly correlated:
regardless of head colour they always occurred together, forming a “behavioural syndrome.” This implies that there is
selection in favour of specific combinations of traits and of head colour in relation to those traits. Selection favours aggression
in red-headed birds and the boldness/ risk-taking behavioural syndrome in black-headed birds.
Based on the text, which choice reflects behaviors UNLIKELY to be exhibited by an individual finch?
A) Returning quickly to feeding after a predator display and failing to approach a novel object
B) Returning slowly to feeding after a predator display and failing to approach a novel object
C) Failing to display aggression and readily approaching a novel object
D) Displaying aggression and being uninterested in exploring new surroundings
170. “Freight Train,” a song Cotten wrote when she was only eleven, was inspired by the locomotive that would rumble by
her window each day. She captured both the sound and speed of its passing as well as the possibility of escape.
Which quotation from “Mabel Osborne” most effectively illustrates that Cotton’s claim?
A. “Freight train, freight train, run so fast / Please don’t tell what train I’m on / They won’t know what route I’m going.”
B. “When I'm dead and in my grave / No more good times here I crave/ Place the stones at my head and feet”
C. “When I die, oh bury me deep / Down at the end of old Chestnut Street / So I can hear old Number Nine”
D. “As she comes rolling by / Tell them all I've gone to sleep”
171. Researchers have shown that, at least in mice, a strong immune response to the flu vaccine relies in part on signals from
intestinal microbes. The findings could help explain variation in the response to the vaccine and suggest ways to maximize its
effectiveness.
The new evidence came out of a curious observation that researchers revealed in a 2011 paper. Bali Pulendran, an
immunologist at Emory University in Atlanta, and colleagues were looking for genetic signatures in the blood of people
injected with the trivalent inactivated influenza vaccine—a mixture of three flu strains. They wanted to know whether the
expression of specific genes in the immune system’s white blood cells correlated with the amount of vaccine- specific
antibodies in the blood—which indicates how strongly a person’s immune system responds to the shot, and how much
protection that person will gain against future infections. In a long list of genes associated with strong vaccine response, the
researchers found an unexpected one: the gene that codes for a protein called toll-like receptor 5 (TLR5).
The author uses the words “curious” and “unexpected” primarily to suggest that Pulendran and his colleagues
A) did not anticipate that the gene coding for TLR5 would be so widespread in the people who received the trivalent vaccine.
B) assumed that the gene coding for TLR5 would not be found in any of the people with weak immune responses to the
trivalent vaccine.

42
https://hocsat.com/
C) were surprised to find a link between the expression of the gene coding for TLR5 and relatively high counts of trivalent
vaccine-specific antibodies.
D) had previously overlooked the connection between the gene coding for TLR5 and changes in white blood cells in response
to the trivalent vaccine.
172. Researchers led by Michael Montague, a postdoc at the Washington University School of Medicine in St. Louis, have now
pinpointed some of them. The scientists started with the genome of a domestic cat—a female Abyssinian—that had been
published in draft form in 2007, then filled in missing sequences and identified genes. They compared the resulting genome
with those of cows, tigers, dogs, and humans.
The analysis, published recently in the Proceedings of the National Academy of Sciences, revealed 281 genes that show signs
of rapid or numerous genetic changes—a hallmark of recent selections—in domestic cats. Some appear to be involved in
hearing and vision, the senses that felines rely on most. Others play a role in fat metabolism and are likely an adaptation to
cats’ highly carnivorous lifestyle.
As presented in the text, Montague’s explanation for the gene changes in domestic cats is based on the assumption that
A) genes associated with particular characteristics in one species are associated with similar characteristics in other species.
B) missing sequences in the genome of one particular breed of cat can be completed by the parallel sequences in the genome
of another breed of cat.
C) the number of genes affected by domestication is greater in some breeds of domestic cats than in other breeds.
D) changes to the cat genome as a result of domestication occurred simultaneously with changes to genomes of other species
associated with sociability.
173. Scientists already know that the Procellarum region is rich in radioactive elements that billions of years ago would have
produced excess heat. The study team theorizes that as this region cooled, the rock would have cracked in geometrical
patterns, like honeycomb patterns seen on Earth in basalt formations, but on a much larger scale. The researchers propose that
these cracks eventually grew into rift valleys, where magma from the Moon’s mantle welled up and pushed apart blocks of
crust. Lava spilled out and paved over the Oceanus Procellarum, creating the dark spot that is seen today. The extra weight of
this dense material would have caused the whole region to sink slightly and form the topographic low that has made the
Procellarum seem so like a basin.
Based on the text, the team led by Andrews-Hanna hypothesized that the Moon would
A) feature geometric patterns on its surface that mirrored subterranean geometric patterns.
B) react to changes in temperature near the surface similarly to how Earth would react.
C) experience higher temperatures in regions where asteroids collided with its surface.
D) give scientists insight into the behavior of the other large moons in the solar system
174. Scientists already know that the Procellarum region is rich in radioactive elements that billions of years ago would have
produced excess heat. They theorize that as this region cooled, the rock would have cracked in geometrical patterns, like
honeycomb patterns seen on Earth in basalt formations, but on a much larger scale. They propose that these cracks eventually
grew into rift valleys, where magma from the Moon’s mantle welled up and pushed apart blocks of crust. Lava spilled out and
paved over the Oceanus Procellarum, creating the dark spot that is seen today. The extra weight of this dense material would
have caused the whole region to sink slightly and form the topographic low that has made the Procellarum seem so like a
basin.
In the text, the scientists propose that which process occured to create the dark spot th that is Oceanus Procellarum?
A) Rigidity of material near the Moon’s surface caused the surface to collapse and form a topographic low.
B) Pressure generated by excess heat from radioactivity caused a change in the composition of the Moon's crust
C) A gradual cooling caused layers beneath the Mourn’s crust to contract.
D) The cracked surface of the region was paved over by material rising up to the surface of the Moon.
175. Scientists already know that the Procellarum region is rich in radioactive elements that billions of years ago would have
produced excess heat. They theorize that as this region cooled, the rock would have cracked in geometrical patterns, like

43
https://hocsat.com/
honeycomb patterns seen on Earth in basalt formations, but on a much larger scale. They propose that these cracks eventually
grew into rift valleys, where magma from the Moon’s mantle welled up and pushed apart blocks of crust. Lava spilled out and
paved over the Oceanus Procellarum, creating the dark spot that is seen today. The extra weight of this dense material would
have caused the whole region to sink slightly and form the topographic low that has made the Procellarum seem so like a
basin.
With the discovery, the Moon joins Earth, Mars, and Venus as Solar system bodies with mapped examples of rifting. There are
also similar features near the south pole of Enceladus, the moon of Saturn that is spewing water into space from cracks in an
ice shell.
The description of other known examples of rifting serves primarily to
A) confirm that Frey is correct in his theory that the Earth and the Moon have identical tectonic mechanisms.
B) reveal that the Procellarum region is unique in the type of rifting it continues to esperience.
C) suggest that the new data collected by GRAIL led to a revised understanding of other celestial bodies.
D) provide additional information that places the conclusions reached by Andrews-Hanna's team in a broader context
176. Scientists have found a nearly square peg underneath a round hole—on the Moon. Several kilometers below Oceanus
Procellarum, the largest dark spot on the Moon’s near side, scientists have discovered a giant rectangle thought to be the
remnants of a geological plumbing system that spilled lava across the Moon about 3.5 billion years ago.
“We’re realizing that the early Moon was a much more dynamic place than we thoughts," says Jeffrey Andrews-Hanna, a
planetary scientist at the Colorado School of Mines in Golden and lead author of a new study of the Procellarum’s geology.
The discovery also casts doubt on the decades—old theory that the circular Procellarum region is a basin, or giant crater,
created when a large asteroid slammed into the Moon. “We don’t expect a basin rim to have corners," Andrews—Hanna says.
Based on the text, it can be inferred that scientists originally identified Oceanus Procellarum as an impact basin as a result of
A) evidence of tectonic and volcanic activity
B) a misintcrpretation of the physical characteristics of the region.
C) unexplained changes observed taking place on the surface of the Moon.
D) evidence that similar patterns in the region shared a condition origin.
177. Scientists have found a nearly square peg underneath a round hole—on the Moon. Several kilometers below Oceanus
Procellarum, the largest dark spot on the Moon’s near side, scientists have discovered a giant rectangle thought to be the
remnants of a geological plumbing system that spilled lava across the Moon about 3.5 billion years ago. The features are
similar to rift valleys on Earth—regions where the crust is cooling, contracting, and ripping apart. Their existence shows that
the Moon, early in its history, experienced tectonic and volcanic activity normally associated with much bigger planets.
The author uses the phrase “geological plumbing system” primarily to
A) explain the meaning of a technical term.
B) indicate the source of a problem.
C) confirm the unexpectedness of a result.
D) provide a description of an internal structure.
178. Selection favours aggression in red-headed birds and the boldness/ risk-taking behavioural syndrome in black-headed birds.
This makes sense when you consider the high risk of predation faced by red-headed birds if they take too many risks and the
need for black-headed birds to find food away from the dominant redheads, which occupy the safest foraging locations.
The author indicates that a possible reason for black-headed finches’ risk-taking behavior is that
A) they are less interested in novelty than are red-headed finches.
B) their conspicuous coloring requires them to be bold when encountering prey.
C) they are more likely to attract predators if they behave more aggressively.
D) they struggle to obtain food at the safer locations favored by red-headed finches.
179. Sergei turned around and looked over his shoulder. “We’ll take scenic route. I want to be first one to show you this great city!”
Emma felt suddenly warm. “No, I have to ...” “Don’t worry. No extra cost for you. Sit back!

44
https://hocsat.com/
Enjoy!” He winked and started the car with a great roar of the engine.
Emma’s fear quickly disappeared as Sergei drove along the Embarcadero toward the bright lights of a place he called
Fisherman’s Wharf. Emma fixed her gaze out the window at the large buildings and the big cars parked along the wide, open
streets—so clean and uncluttered.
According to the text, Sergei can most likely be characterized as
A) generous and reassuring.
B) proud and boisterous.
C) curious and innocent.
D) poised and serene
180. She was persuaded to believe the engagement a wrong thing—indiscreet, improper, hardly capable of success, and not
deserving it. But it was not a merely selfish caution, under which she acted, in putting an end to it. Had she not imagined
herself consulting his good, even more than her own, she could hardly have given him up. The belief of being prudent and self-
denying principally for his advantage, was her chief consolation, under the misery of a parting—a final parting; and every
consolation was required, for she had to encounter all the additional pain of opinions, on his side, totally unconvinced and
unbending, and of his feeling himself ill-used by so forced a relinquishment. Captain Wentworth had left the country in
consequence.
The text indicates that Anne's termination of her engagement to Captain Wentworth was the result of
A) a decision based largely on her perception of hip poorest interests.
B) an acknowledgment of the force other fathers disapproval.
C) an awareness of her diminishing feelings for Captain Wentworth.
D) an outcome that Captain Wentworth had not anticipated.
181. Live thy Life,
Young and old,
Like yon oak,
Bright in spring,
Living gold;
Summer-rich
Then: and then
Autumn-changed,
Soberer-hued
Gold again.
All his leaves
Fall at length,
Look, he stands,
Trunk and bough,
Naked strength.
“The Oak* by Lord Alfred Tennyson
In this poem, the seasons represent different
(A) kinds of trees
(B) times of day
(C) stages of life
(D) styles of dress
182. Since disfluency shows that a speaker is thinking carefully about what she is about to say, they provide useful information to
listeners, cueing them to focus attention on upcoming content thats likely to be meaty. One famous example comes from the
movie Jurassic Park. When Jeff Goldblums character says, "I’m, I’m simply saying that life, uh ... finds a way;‘ the

45
https://hocsat.com/
disfiuencies emphasize that he's coming to grips with something not easy to explain—an idea that turns out to be a key part of
the movie.
Based on the text, which choice best identifies one way that disfluency inadvertently assists communication?
A) By signifying the speaker's conscious focus on specific points, disfluency emphasizes which aspects of an utterance are apt
to be significant.
B) By moderating the pace at which the speaker introduces details, disfluency enhances listeners’ recall of a narrative as a
whole.
C) By recapturing the listeners attention as it wanes over the course of a narrative, disfluency ensures general comprehension.
D) By conveying the speaker's relatability, disfluency renders an utterance more relevant to the listener than it would
otherwise be.
183. So we began an ambitious program at visible and near-infrared wavelengths as a natural successor to HUDF: the Cosmic
Assembly Near-infrared Deep Extragalactic Legacy Survey (CANDELS), pronounced “candles.” We designed CANDELS
primarily to ___________ the first one-third of galaxy evolution.
Which choice completes the text with the most logical and precise word or phrase?
A) produce.
B) define.
C) record.
D) construct.
184. So why are cats still a bit wilder than our other favorite domesticate, the dog? Co-author William Murphy, a geneticist at
Texas A&M University, College Station, says the cat genome appears to have undergone less intense and more recent
evolutionary pressure than that of dogs; that’s not surprising, considering that dogs may have lived with us for up to 30,000
years. “Cats were not selected for a purpose like dogs and other domesticates,” Murphy speculates.
The author most likely includes the quotation in the last sentence of the text to
A) offer an alternative to generally accepted beliefs about the domestication of cats.
B) suggest a potential explanation for the distinct way in which domestic cats developed.
C) provide a possible reason for the continual changes to the cat genome.
D) qualify previously discussed findings about domestic animals.
185. Solow growth model, named for Robert Solow, a Nobel-prize winning economist posits that poorer countries should generally
invest more and grow faster than rich ones. Central banks and other big economic institutions use far more complicated
formulas, often grouped under the bewildering label of “dynamic stochastic general equilibrium” (DSGE) models. These try to
anticipate the ups and downs of big economies by modelling the behaviour of individual households and firms. DSGE models,
for all their complexity, are typically built around oversimplifications of how markets function and people behave.
The author most strongly suggests that some large economic institutions have relied on models that assume that
A) the behavior of households and firms has a larger effect on economies than they truly do.
B) data-based forecasts evaluate more economic variables than they truly do.
C) statistical problems with data-based forecasts are worse than they truly are.
D) individuals and companies act in ways that are more predictable than they truly are.
186. Some one has said that a truth once spoken can never be _________. It goes on and on, and no one can set a limit to its ever-
widening influence. But if it were possible to obliterate every word written or spoken in defense of the principles set forth in
the Declaration of Independence, a war of conquest would still leave its legacy of perpetual hatred, for it was God himself who
placed in every human heart the love of liberty. He never made a race of people so low in the scale of civilization or
intelligence that it would welcome a foreign master.
Which choice completes the text with the most logical and precise word or phrase?
A) repeated.
B) retracted.

46
https://hocsat.com/
C) rejected.
D) remembered.
187. Stomach churning, Mr. Kohlah was absorbed in watching the growth of development in the hills. His friends and he agreed it
was a malevolent growth. The possibility of increased business at the General Store was no consolation. All his senses were
being assaulted by the invasion. The noxious exhaust from lorries was searing his nostrils he told Mrs. Kohlah, and the ugly
throbbing of their engines was ripping his eardrums.
Whenever he turned, he began to see the spread of shacks and shanties. It reminded him of the rapidity with which the mange
had overtaken his favourite dog. The destitute encampments scratched away at the hillsides, the people drawn from every
direction by stories of construction and wealth and employment.
The author's use of the words "assaulted," "invasion," and "encampments," primarily serves to
A) illustrate the villagers' aggressive response to development in the region.
B) compare the effects of the new road to a military siege.
C) expose the negative effect construction in the area has had on the store.
D) mock the notion that things in the village can remain as they were.
188. Subhash wondered if his placid nature was regarded as a lack of inventiveness, perhaps even a failing, in his parents’ eyes. His
parents did not have to worry about him and yet they did not favor him. It became his mission to obey them, given that it
wasn’t possible to surprise or impress them. That was what Udayan did.
It can reasonably be inferred from the text that Subhash perceives his parents as
A) mistaking his self-restraint for apathy.
B) preferring his brother to him.
C) resenting him as a reflection of their own weakness.
D) underestimating the effort that obeying them requires
189. Such opposition, as these feelings produced, was more than Anne could combat. Young and gentle as she was, it might yet
have been possible to withstand her father's ill-will, though unsoftened by one kind word or look on the part of her sister; but
Lady Russell, whom she had always loved and relied on, could not, with such steadiness of opinion, and such tenderness of
manner, be continually advising her in vain. She was persuaded to believe the engagement a wrong thing—indiscreet,
improper, hardly capable of success, and not deserving it.
The sentence in underlined lines serves mainly to reveal that
A) Anne held her sister's opinions about romance in high regard.
B) Lady Russell took advantage of her relationship with Anne.
C) Anne was more influenced by Lady Russell than by her father.
D) Lady Russell believed that Annes engagement was improper.
190. Swiftly, on limitless foundations, the United States . . . are founding a literature. It is all as well done, in my opinion, as could
be practicable. Each element here is in condition. Every day I go among the people of Manhattan Island, Brooklyn, and other
cities, and among the young men, to discover the spirit of them, and to refresh myself. These are to be attended to; I am myself
more drawn here than to those authors, publishers, importations, reprints, and so forth. I pass coolly through those,
understanding them perfectly well, and that they do the indispensable service, outside of men like me, which nothing else
could do. In poems, the young men of The States shall be represented, for they out—rival the best of the rest of the earth.
In describing his own activity, Whitman suggests that the components of American literature can more readily be found in
A) New York City than in other cities in the nation.
B) the work of common laborers than in that of educated professionals.
C) the genre of poetry than in other literary genres.
D) the daily lives of city dwellers than in published literature.
191. Swiftly, on limitless foundations, the United States … are founding a literature. It is all as well done, in my opinion, as could
be practicable. Each element here is in condition. Every day I go among the people of Manhattan Island, Brooklyn, and other

47
https://hocsat.com/
cities, and among the young men, to discover the spirit of them, and to refresh myself. These are to be attended to; I am myself
more _________ here than to those authors, publishers, importations, reprints, and so forth. I pass coolly through those,
understanding them perfectly well, and that they do the indispensable service, outside of men like me, which nothing else
could do.
Which choice completes the text with the most logical and precise word or phrase?
A) provoked
B) extracted
C) dragged
D) attracted
192. Taiki Adachi and his colleagues developed a new type of accelerometer to find out. When worn by an elephant seal, the device
can monitor cyclic patterns in speed and count each surge forward as one stroke of the flippers. By also tracking depth and
swimming angle, the device can constantly measure the seal’s rate of strokes per distance traveled. Seals that make more
strokes are working harder.
Which statement about the number of swimming stroke taken by northern elephant seals is supported by text?
A) A greater number of strokes signifies a greater amount of energy expended.
B) A greater number of strokes signifies a more pronounced swimming angel.
C) A lesser number of strokes signifies a potential for deeper diving depth.
D) A lesser number of strokes signifies a more urgent need for nutritional supplementation
193. Ten years ago the number of taxpayers in Greenspace County was slightly greater than the number of registered voters. The
number of taxpayers has doubled over the last ten years, while the number of registered voters has increased, but at a lower
rate than has the number of taxpayers.
Which of the following must be true in Greenspace County if the statements above are true?
(A) The number of taxpayers is now smaller than the number of registered voters.
(B) Everyone who is a registered voter is also a taxpayer.
(C) The proportion of registered voters to taxpayers has increased over the last ten years.
(D) The proportion of registered voters to taxpayers has decreased over the last ten years.
194. Text 1
At least some of those species of earthworm had the habit of depositing their castings above ground. A worm would back
tail—first out of its burrow and unload a neat castellated pile around the entrance. As a result, Darwin recognized, soil from a
foot or more underground was steadily being carried up to the surface. In many parts of England, he figured, the worm
population swallowed and brought up ten tons of earth each year on each acre of land. Earthworms therefore were not only
creating the planet’s thin layer of fertile soil; they were also constantly turning it inside out. No wonder Darwin concluded:
“Worms have played a more important part in the history of the world than most persons would at first suppose.”
Text 2
In the 1990s, biologist Cindy Hale took a field trip to northern Minnesota. There she saw a changed landscape. Gone were
forest floor plants like ferns and wildflowers and understory plants like bushes and tree saplings that make up the middle
height of the forest. She and other ecologists on the field trip were given a surprising reason for the loss of the plants and the
ecosystem they sustained: invasive earthworms.
Which choice identifies a central difference in the ecological focus of the two passages?
A) Text 1 surveys the changing impact of a species over the course of its existence, whereas Text 2 focuses on a species’
impact in the present only.
B) Text 1 mainly portrays a creature’s role in gradually shaping the environment, whereas Text 2 examines ways in which a
creature can cause environmental destabilization.
C) Text 1 considers interactions among multiple species in an ecosystem, whereas Text 2 concentrates on the role that a single
species has on its ecosystem.

48
https://hocsat.com/
D) Text 1 limits its discussion to habitats with no human presence, whereas Text 2 explores the relationship between human
societies and the natural world.
195. Text 1
Earth worm swallowed dead plant material for its sustenance, and it swallowed almost anything else in its path (including tiny
rock particles) as it burrowed. The rock particles were smashed even finer in the worm’s gizzard, mixed with the plant material
and the digestive juices in its gut, and passed out behind in the form of“castings.” The castings contained enough natural glue
to give them a nice crumb structure, characteristic of good soil, and were also biochemically ideal for nurturing vegetation.
Text 2
In one recent study, scientists looked at how invasive earthworms have affected a type of litter- dwelling mite. Mites help
break down a forest floor’s duff and spread fungus spores, the tiny reproductive units similar to seeds that give rise to more
fungi. Today, more than 100,000 mites of more than 100 species may occupy each square meter of northern forest soil. That
may sound like a lot, but this study showed that in soil free of invasive earthworms, the mites seem to fare better. They were
between 72 and 1,210 times more abundant and the number of mite species was one to two times higher.
The author of Text 2 would most likely describe the biological processes discussed in the underlined portion as
A) reflective of the changes that a species typically undergoes when transplanted to an unfamiliar environment.
B) potentially harmful to ecosystems to which the organisms that perform those processes are not native.
C) jeopardized by the presence of competitor species that perform equivalent processes.
D) frequently misunderstood as conferring an advantage to all the organisms in an ecosystem.
196. Text 1
In the courts, impartiality is required, and political compromise has no role at all. Judges are supposed to be reflective,
considering the controversy before them in light of the broader legal schemes, Constitutional and otherwise, which guide the
country. Decisions traditionally are justified by opinions announcing reasoning derived from earlier cases and established
principles; raw political power is never a sufficient justification for any judicial decision. Constitutional rights should never be
compromised by the courts in the name of expediency.
Text 2
Only in recent decades have party politics infiltrated the marble halls of the Supreme Court, and only in the past few years
have they become the best predictor of its major ruling. The Supreme Court has never divided along partisan lines as neatly as
it does today. A high court that splits into ideological camps while purporting to provide "equal justice under law" calls into
question its very legitimacy.
Both authors would consider the integrity of a court ruling to be compromised under which condition?
A) The ruling invokes legislative and executive precedent as well as judicial precedent.
B) Judges defend a ruling they have issued by commenting on it outside of the courtroom.
C) Political tendencies inform the legal reasoning behind the ruling.
D) The values of the judges differ from those embodied by the government.
197. Text 1
It can never be the greatest good for the greatest number to deny the equal moral worth of a fellow human being. Similarly,
when the interests of individuals clash, there cannot be any danger of predisposition by the court if each litigant is to be
confident that he has received equal treatment. Before the bar, all men and women must stand equal, with their claims resolved
solely on the strength of legal principles.
Text 2
Indeed, new research by three political scientists shows just how avidly the justices go to bat for causes they identify with.
Contrary to stereotypes about the relative friendliness of conservatives or liberals to free-speech claims generally, Ms Epstein
and her co-authors found that the justices are "opportunistic free speechers." Some principle might be found to account for the
suspicious patterns in their votes, but the evidence looks pretty damning. Justices’ votes "are neither reflexively pro- or anti-
the first amendment"; they are, instead, for or against "the speaker’s ideological enclave."

49
https://hocsat.com/
How would the atuhor of text 2 most likely respond to the position underlined in text 1?
A) He would argue that although casual observation of Supreme Court ruling debunks Marshall’s claim, examination of them
by legal scholars confirms it.
B) He would consider Marshall’s description true for the majority of Supreme Court ruling but misrepresentative of rulings in
free-speech cases.
C) He would contend that Marshall’s assessment was an unusual one in the past but has grown increasingly mainstream in the
current political climate.
D) He would assert that Marshall’s sentiment is contradicted by judge’s sympathetic treatment of litigants whose beliefs they
endorse.
198. Text 1
The question of why zebras have stripes has puzzled scientists—including Darwin—for over a century, leading to five main
hypotheses: that the stripes repel insects, provide camouflage, confuse predators, reduce body temperature, or help the animals
interact socially.
For the first time, scientists played all of these theories against each other in a statistical model—and the result was pretty
much, well, black and white.
“We found again and again and again [that] the only factor which is highly associated with striping is to ban biting flies,” said
study leader Tim Caro, a biologist at the University of California, Davis.
Text 2
Researchers based at the University of California, Los Angeles (UCLA) have produced one of the most comprehensive zebra
stripe studies yet by examining how 29 different environmental variables influence the stripe styles of plains zebras at 16
different sites from south to central Africa.
The scientists found that the definition of stripes along a zebra’s back most closely correlated with temperature and
precipitation in a zebra’s environment, and did not correlate with the prevalence of lions or tsetse flies in the region. These
findings suggest that torso stripes may do more to help zebras regulate their body temperature than to avoid predators and
tsetse flies.
The primary purpose of both texts is to consider studies that
A) describe the process by which stripes became widespread among zebra species.
B) dispute a common misconception regarding zebras’ stripes.
C) explore the likely benefit that zebras derive from their stripes.
D) compare zebras’ stripes to coloration in other animal species.
199. Text 1
We found again and again and again [that] the only factor which is highly associated with striping is to ban biting flies,” said
study leader Tim Caro, a biologist at the University of California, Davis.
For the study, Caro and colleagues collected data from a vast range of sources, including museum collections and historical
maps.
Text 2
Researchers based at the University of California, Los Angeles (UCLA) have produced one of the most comprehensive zebra
stripe studies yet by examining how 29 different environmental variables influence the stripe styles of plains zebras at 16
different sites from south to central Africa.
The scientists found that the definition of stripes along a zebra’s back most closely correlated with temperature and
precipitation in a zebra’s environment, and did not correlate with the prevalence of lions or tsetse flies in the region. These
findings suggest that torso stripes may do more to help zebras regulate their body temperature than to avoid predators and
tsetse flies
Researchers in both texts defend their conclusions by relying on
A) several experiments that simulate conditions in nature.
B) field observations over the course of several months.

50
https://hocsat.com/
C) data collected from various museums and maps.
D) observed correlations among multiple variables.
200. Text 1
Whenever deception, falsehood, and fraud creep in they undermine the whole structure. They damage the whole art. The
efforts of the Government to secure correct labels, fair trade practices, and equal opportunity for all our inhabitants is
fundamentally an effort to get the truth into business. The Government can do much in this direction by setting up correct
standards, but all its efforts will fail unless it has the loyal support of the business men of the Nation. If our commercial life is
to be clean and wholesome and permanent in the last resort, it will be because those who are engaged in it are determined to
make it so.
Text 2
The great bulk of the things which we consumers buy are not reviewed by any impartial testing body. Most of them advance
upon us from behind a great smoke screen of advertising. Given time enough, and trial and error enough, quality will in many
cases make itself felt. But consider the waste of this trial and error method as against a permanent source to which we might
turn for the results of scientific tests and the setting of impartial standards.
The authors of both texts would most likely agree with which statement about deceptive advertisements?
A) They are generally more effective than truthful advertisements.
B) They are more common in certain media than in others.
C) They tend to inspire public cynicism about the advertising industry.
D) They are unlikely to mislead the public indefinitely.
201. That designation of evil is one of the two prevailing narratives concerning genetically engineered foods. Opponents of
genetically modified organisms (GMOs) tell the story that “Franker” organisms are a new technology that _________ known
and unknowable dangers to human health, the environment, and society at large.
Which choice completes the text with the most logical and precise word or phrase?
A) arranges.
B) presents.
C) questions.
D) imitates.
202. That designation of evil is one of the two prevailing narratives concerning genetically engineered foods. Opponents of
genetically modified organisms (GMOs) tell the story that “Franker” organisms are a new technology that _________ known
and unknowable dangers to human health, the environment, and society at large.
Which choice completes the text with the most logical and precise word or phrase?
A) arranges.
B) presents.
C) questions.
D) imitates.
203. That huge English flow, so sweet, so undeniable, has done incalculable good here, and is to be spoken of for its own sake with
generous praise and with gratitude. Yet the price The States have had to lie under for the same has not been a small price.
Payment prevails; a nation can never take the issues of the needs of other nations for nothing.
The author makes which assertion about how the English literary tradition should be regarded in the United States?
A) Critics should view it as antiquated and obsolete.
B) Writers should imitate its style but write about American themes.
C) Readers should consider it equal to Western classical literature.
D) Americans should honor and be thankful for it.
204. That year, when Maneck came home for the holidays, he was puzzled (and later alarmed) to discover his father perpetually
irritable. They found it impossible to get through the day without quarrelling, breaking into argument even in the presence of

51
https://hocsat.com/
customers.
"What's the matter with him?" Maneck asked his mother. "When I'm here, he ignores me or fights with me. When I'm at
school, he writes letters saying how much he misses me."
Which choice best describes how Maneck characterizes his father's attitude toward him?
A) His father is relieved to have Maneck back in the village but preoccupied with the new road.
B) His father is affectionate when Maneck is away but cold when Maneck is home.
C) His father is regretful of having sent Maneck away to boarding school.
D) His father is perplexed and troubled by how different Maneck seems after his return.
205. The ability to __________ subtle variations in ambient CO2, concentration is well established among moths. CO2, receptor
cells are located in a sensory organ, the Line labial-palp pit organ (LPO). Morphological studies have shown that this organ
contains up to 2,000 receptor cells, and physiological experiments have revealed that those sensory cells respond specifically
to CO₂, with high sensitivity. For most species of moths, however, the roles of sensory information about ambient CO2, are
unclear.
Which choice completes the text with the most logical and precise word or phrase?
A) perceive.
B) understand.
C) anticipate.
D) touch.
206. The author implies that the researchers' conclusion would have been less valid if the
A) texts that were studied included words with positive connotations.
B) iterary misery index was defined by the frequency of just four words.
C) books analyzed discussed the economic fluctuations of a particular decade.
D) study was made up of texts written in a single language.
207. The authors begin by unfolding a new map. Using data compiled by BirdLife International, they show that the global
distribution of cooperatively breeding birds overlaps strikingly with that of brood parasites. This overlap need not __________
a causal relationship: The same unpredictable environments that favor cooperation could also favor alternative breeding
strategies such as parasitism.
Which choice completes the text with the most logical and precise word or phrase?
A) consider.
B) indicate.
C) speculate.
D) reproduce.
208. The authors begin by unfolding a new map. Using data compiled by BirdLife International, they show that the global
distribution of cooperatively breeding birds overlaps strikingly with that of brood parasites. This overlap need not reflect a
causal relationship: The same unpredictable environments that ___________ cooperation could also _________ alternative
breeding strategies such as parasitism.
Which choice completes the text with the most logical and precise word or phrase?
A) facilitate.
B) resemble.
C) indulge.
D) commend.
209. The chemical adenosine is released by brain cells when those cells are active. Adenosine then binds to more and more sites oii
cells in certain areas of the brain, as the total amount released gradually increases during wakefulness. During sleep, the
number of sites to which adenosine is bound decreases. Some researchers have hypothesized that it is the cumulative binding
of adenosine to a large number of sites that causes the onset of sleep.

52
https://hocsat.com/
Which of the following, if true, provides the most support for the researchers' hypothesis?
(A) Even after long periods of sleep when adenosine is at its lowest concentration in the brain, the number of brain cells bound
with adenosine remains very large.
(B) Caffeine, which has the effect of making people remain wakeful, is known to interfere with the binding of adenosine to
sites on brain cells.
(C) Besides binding to sites in the brain, adenosine is known to be involved in biochemical reactions throughout the body.
(D) Some areas of the brain that are relatively inactive nonetheless release some adenosine
210. The Cherokee people and Government have given every assurance in their power of their sympathy and friendship for the
people of Arkansas and of other Confederate States, unless it be in not voluntarily ____________ an attitude of hostility
toward the Government of the United States, with whom their treaties exist and from whom they are not experiencing any new
burdens or exactions. That I cannot advise them to do, and hope that their good faith in adhering to the requirements of their
treaties and of their friendship for all the whites will be manifested by strict observance of the neutrality enjoined.
Which choice completes the text with the most logical and precise word or phrase?
A) adopting.
B) pretending.
C) borrowing.
D) presuming.
211. The coffee-berry borer is a pesky beetle. It is thought to destroy $500 million worth of unpicked coffee beans a year, thus
diminishing the incomes of some 20 million farmers. The borer spends most of its life as a larva, buried inside a coffee berry,
feeding on the beans within. To do so, it has to defy the toxic effects of caffeine. This is a substance which, though pleasing to
people, is fatal to insects—except, for reasons hitherto unknown, to the coffee-berry borer.
According to the text, coffee-berry borers are a problem for coffee farmers because they
A) render coffee beans toxic.
B) infect coffee plants with bacteria.
C) consume the coffee beans.
D) weaken the coffee plant’s root system.
212. The coffee-berry borer is a pesky beetle. It is thought to destroy $500m-worth of unpicked coffee beans a year, thus
__________ the incomes of some 20m farmers. The borer spends most of its life as a larva, buried inside a coffee berry,
feeding on the beans within. To do so, it has to defy the toxic effects of caffeine. This is a substance which, though pleasing to
people, is fatal to insects—except, for reasons hitherto unknown, to the coffee-berry borer
Which choice completes the text with the most logical and precise word or phrase?
A) belittling
B) reducing
C) discrediting.
D) underestimating
213. The conference where US Supreme Court justices start deciding a case is purposely structured. “In the conference, we go
around the table in order of seniority, from the chief justice down to the most recent appointment,” Justice Stephen Breyer
explained, “and everybody speaks once before anybody speaks twice.” This ensures that every opinion is represented. “Each
person might spend five minutes per case . . . They’re trying to explain their reasons for which direction they’re leaning. And
everybody writes down what everybody else says. And then there’ll be some discussion back and forth afterwards. And on the
basis of that discussion—which is a preliminary discussion—it’s fairly clear how the Court is likely to break down.”
According to the text, preliminary conferences for US Supreme Court cases are structured with which primary aim in mind?
A) Allowing each justice to present his or her position
B) Privileging the opinions of more recently appointed justices

53
https://hocsat.com/
C) Reaching a majority decision by minimizing debate among justices
D) Compiling a written transcript of the arguments advanced by each justice
214. The conference where US Supreme Court justices start deciding a case is purposely structured. “In the conference, we go
around the table in order of seniority, from the chief justice down to the most recent appointment,” Justices Stephen Breyer
explained, “and everybody speaks once before anybody speaks twice.” This ensures that every opinion is ___________. “Each
person might spend five minutes per case…They’re trying to explain their reasons for which direction they’re leaning. And
everybody writes down what everybody else says. And then there’ll be some discussion back and forth afterwards. And on the
basis of that discussion—which is a preliminary discussion—it’s fairly clear how the Court is likely to break down.”
Which choice completes the text with the most logical and precise word or phrase?
A) served
B) expressed
C) portrayed
D) symbolized
215. The dark veil enshrouding the ancient pictures had not yet wholly passed away from before them; but Tchartkoff already saw
something in them, though in private he did not agree with the professor that the secrets of the old masters are irremediably
lost to us. It seemed to him that the nineteenth century had improved upon them considerably, that the delineation of nature
was more clear, more vivid, more close.
According to the text, one point of disagreement between Tchartkoff and his professor concerns whether
A) making money from selling paintings destroys artistic integrity.
B) fashionable artists are capable of making enough money from their art to support themselves.
C) nineteenth-century painters had been able to expand on the insights of the old masters.
D) nonprofessional painters are capable of producing serious artworks.
216. The environmental consequences of engineered genes escaping into the wild are less clear. But while fallout can be hard to
predict, the odds of such escapes actually happening can often be evaluated. With the Food and Drug Administration’s recent
approval of GM salmon, for example, scientists agree that there is a slim possibility that escapees could harm native fish
populations; that risk could be curtailed, however, with strict oversight about where and how such fish are farmed.
The author makes which of the following claims about GM salmon?
A) The fishing industry cannot be trusted to prevent GM salmon from escaping.
B) New research has led to the development of environmentally safe GM salmon.
C) The current inadequate regulations surrounding GM salmon fail to protect native fish.
D) The potential dangers associated with GM salmon can be anticipated and managed.
217. The great bulk of the things which we consumers buy are not reviewed by any impartial testing body. Most of them advance
upon us from behind a great smoke screen of advertising. Given time enough, and trial and error enough, quality will in many
cases make itself felt. But consider the waste of this trial and error method as against a permanent source to which we might
turn for the results of scientific tests and the setting of impartial standards.
According to the text, consumers generally learn about the merits of a product by
A) using it and observing the results over time.
B) reading reviews written by journalists.
C) assessing reports published by scientists.
D) engaging in conversations with their friends
218. The idea that plants have developed a subterranean internet, which they use to raise the alarm when danger threatens, sounds
like science fiction. But David Johnson of the University of Aberdeen believes he has shown that just such an internet, with
fungal hyphae [the branching filaments that make up a fungus’s body] standing in for local Wi-Fi, alerts beanstalks to danger
if one of their neighbours is attacked by aphids.
The text introduces the topic mainly by

54
https://hocsat.com/
A) indicating that communication among plants is more widespread than is recognized.
B) emphasizing the complexity of plant and parasite interactions.
C) using an analogy to show how communication among plants might occur.
D) providing a rationale for an unorthodox research study on plants.
219. The judiciary operates under a premise of neutrality rather than partisanship. Federal judges are insulated, as much as possible,
from political pressure which might interfere with principled decision making. Article II judges have life tenure and are free
from threats of economic retaliation for unpopular decisions. In addition, we cannot have a personal stake in the outcome of
any case before us, and the Code of Judicial Conduct cautions us to avoid even the appearance of impropriety.
It can reasonably be inferred that the author believes the purpose of the federal judiciary would be undermined if it were
altered in which way?
A) Officials in other branches of government were forbidden from discussing federal court cases with the press.
B) The ethical standards of the judiciary were determined by its members rather than by other branches.
C) Judges were subject to reappointment or dismissal by elected officials, based on a review of their rulings.
D) Court ruling were narrowly tailored to the specifics each case, avoiding fundamental political and philosophical questions
220. The last members of a now-extinct species of a European wild deer called the giant deer lived in Ireland about 16,000 years
ago. Prehistoric cave paintings in France depict this animal as having a large hump on its back. Fossils of this animal,
however, do not show any hump. Nevertheless, there is no reason to conclude that the cave paintings are therefore inaccurate
in this regard, since ___________
Which of the following most logically completes the argument?
(A) some prehistoric cave paintings in France also depict other animals as having a hump
(B) fossils of the giant deer are much more common in Ireland than in France
(C) animal humps are composed of fatty tissue, which does not fossilize
(D) the cave paintings of the giant deer were painted well before 16,000 years ago
221. The lists of ready-made literature which America inherits by the mighty inheritance of the English language all the
____________ repertoire of traditions, poems, histories, metaphysics, plays, classics, translations, have made, and still
continue, magnificent preparations for that other plainly signified literature, to be our own, to be electric, fresh, lusty, to
express the full-sized body, male and female—to give the modern meanings of things, to grow up beautiful, lasting,
commensurate with America, with all the passions of home, with the inimitable sympathies of having been boys and girls
together, and of parents who were with our parents.
Which choice completes the text with the most logical and precise word or phrase?
A) manifestly affluent
B) abundantly supplied
C) gaudily impressive
D) deeply colored
222. The lists of ready-made literature which America inherits by the mighty inheritance of the English language—all the rich
repertoire of traditions, poems, histories, metaphysics, plays, classics, translations, have made, and still continue, magnificent
preparations for that other plainly signified literature, to be our own, to be electric, fresh, lusty, to express the full-sized body,
male and female—to give the modern meanings of things, to grow up beautiful, lasting, commensurate with America, with all
the passions of home, with the inimitable sympathies of having been boys and girls together, and of parents who were with our
parents.
The primary purpose of the paragraph is to argue that an older literary tradition is
A) a repository containing everything necessary for creating a new literature.
B) an important asset for a nation that celebrates individual freedom.
C) a valuable basis for a new and singular development.
D) a relic with historical significance but with no contemporary relevance.

55
https://hocsat.com/
223. The most intriguing findings came when Montague’s team sequenced the genomes of 22 domestic cats—representing a wide
variety of breeds and locations—and compared them with the genomes of two Near Eastern and two European wildcats. The
researchers uncovered at least 13 genes that changed as cats morphed from feral to friendly. Some of these, based on previous
studies of knockout mice [genetically engineered mice], seem to play a role in cognition, including fear responses and the
ability to learn new behaviors when given food rewards.“That jibes with what we know about the domestication of cats,”
Montague says, “because they would have needed to become less fearful of new locations and individuals, and the promise of
food would have kept them sticking around.”
Which choice best describes how Montague’s team achieved the results?
A) The team sequenced the genomes of twenty-two breeds of domestic cats from around the world, compared those genomes
to one another, and identified the genes that appear to have changed as a result of breeding.
B) The team used a previously published genome sequence of a domestic cat to locate the genes associated with hearing and
vision, compared those genes to the same genes in two species of wildcats, and identified the point in time at which those
genes appear to have diverged.
C) The team sequenced the genomes of several breeds of geographically diverse domestic cats, compared those genomes to the
genomes of four wildcats from two locations, and identified the genes that appear to have changed due to domestication.
D) The team sequenced the genomes of domestic cats from two locations, compared those genomes to the genomes of wildcats
from the same locations, and identified which genes appear to be geographically specific.
224. The new evidence came out of a curious observation that researchers revealed in a 2011 paper. Bali Pulendran, an
immunologist at Emory University in Atlanta, and colleagues were looking for genetic signatures in the blood of people
injected with the trivalent inactivated influenza vaccine—a mixture of three flu strains. They wanted to know whether the
expression of specific genes in the immune system’s white blood cells correlated with the amount of vaccine- specific
antibodies in the blood—which indicates how strongly a person’s immune system responds to the shot, and how much
protection that person will gain against future infections. In a long list of genes associated with strong vaccine response, the
researchers found an unexpected one: the gene that codes for a protein called toll-like receptor 5 (TLR5).
According to the text, which statement best explains the hypothesis that Pulendran’s group tested in their experiment with
mice?
A) B cells must receive signals from intestinal bacteria to begin producing vaccine-specific antibodies.
B) Vaccines containing active viruses must be accompanied by adjuvants to generate a strong immune response.
C) The effectiveness of vaccines at stimulating flagellin activity depends on the presence of the gene that codes for TLR5.
D) The strength of the immune response following vaccination is related to the white blood cells’ detection of signals from
intestinal bacteria.
225. The phenomenon of false memories is common to everybody—the party you’re certain you attended in high school, say, when
you were actually home with the flu, but so many people have told you about it over the years that it’s made its way into your
own memory cache. False memories can sometimes be a mere ___________, but other times they have real implications.
Innocent people have gone to jail when well-intentioned eyewitnesses testify to events that actually unfolded an entirely
different way.
Which choice completes the text with the most logical and precise word or phrase?
A) concern.
B) question.
C) oddity.
D) wonder.
226. The phenomenon of false memories is common to everybody—the party you’re certain you attended in high school, say, when
you were actually home with the flu, but so many people have told you about it over the years that it’s made its way into your
own memory cache. False memories can sometimes be a mere curiosity, but other times they have real implications. Innocent
people have gone to jail when well-intentioned eyewitnesses testify to events that actually unfolded an entirely different way.

56
https://hocsat.com/
Which statement about false memories can reasonably be inferred from the text?
A) They can interfere with a person’s deductive reasoning ability.
B) They correlate with attempts to remember large amounts of information.
C) They are more commonly associated with events in the distant rather than the recent past.
D) They can have consequences that are genuinely damaging.
227. The picture drawn from decades of research is out of sync with many common public perceptions. While unforeseeable health
issues are often at the forefront of public concern, foods containing GMOs have been on grocery shelves for more than 20
years. Piles of evidence suggest that eating GMOs is no riskier than eating conventional foods. Effects on the environment are
more mixed. Some of the problems that have arisen, such as the uptick in the use of certain herbicides, are more about farming
practices than about dangers inherent to GM technology; the same problems arise with conventional, non-GM crops.
Which choice best describes the author’s perspective on the consumption of genetically modified foods?
A) There is little reason to believe that consuming GMOs is harmful to humans.
B) GM foods require further study before scientists can determine conclusively that they are healthful.
C) It is safe to use GM crops to feed livestock, but it is not safe for humans to consume them directly.
D) Years of research have demonstrated that GMOs should not be part of the human diet.
228. The play was one of those drawing-room concoctions in which charmingly overdressed ladies and gentlemen suffer the pangs
of love and jealousy amid gilded surroundings. Such bon-mots are ever enticing to those who have all their days longed for
such material surroundings and have never had them gratified. They have the charm of showing suffering under ideal
conditions. Who would not grieve upon a gilded chair? Who would not suffer amid perfumed tapestries, cushioned furniture,
and liveried servants? Grief under such circumstances becomes an enticing thing. Carrie longed to be of it. She wanted to take
her sufferings, whatever they were, in such a world, or failing that, at least to simulate them under such charming conditions
upon the stage. So affected was her mind by what she had seen, that the play now seemed an extraordinarily beautiful thing.
She was soon lost in the world it represented, and wished that she might never return.
According to the text, as Carrie views the play, she envies which aspect of the characters’ lives?
A) Their lack of everyday problems
B) Their ability to forget the past easily and focus on the present
C) Their opportunity to bear their sufferings in fortunate circumstances
D) Their open indifference to material possessions
229. The primary line of evidence was the fusion of the skull bones. As animals age, the various bones that make up their skulls
fuse along sutures, and the degree to which the bones have fused can sometimes be used to roughly determine age. Since all
the skull bones in the Gilmore skull appeared to be fused, Bakker and colleagues stated, the tyrannosaur must have been a
small adult and therefore ___________ from the bigger, bulkier Tyrannosaurus rex. Appropriately, they called the
hypothesized animal Nanotyrannus.
Which choice completes the text with the most logical and precise word or phrase?
A) noble.
B) different.
C) obvious.
D) unusual.
230. The researchers found that when wasps spent an average of 3½ minutes in the chamber containing volatiles that beanstalks
release and 1½ in the other chamber. Aphids, conversely, spent 1¾ minutes in the volatiles’ chamber and 3¼ in the control. In
other words, the volatiles from an infested plant attract wasps and repel aphids.
Crucially, the team got the same result in the case of uninfested plants that had been in uninterrupted hyphal contact with the
infested one, but had had root contact blocked.
According to the text, which factor is most likely responsible for aphids’ attraction to some of the uninfested plants in the
experiment?

57
https://hocsat.com/
A) The plants were unable to receive distress signals from infested plants through hyphal contact.
B) The plants had emitted a chemical that repelled the wasps that were attracted to infested plants.
C) The plants had developed hyphal connections with the fungi.
D) The plants’ root systems had become compromised by the aphids.
231. The researchers grew corn seedlings of a common variety, called B73, and exposed some to caterpillars. They then seeded
them with aphids and counted the number of offspring that the aphids produced on pristine plants, compared to previously
nibbled ones. The aphids consistently produced more offspring on corn that had been pre-chewed by caterpillars.
By describing the corn plants as "pristine", the author most likely means that the plants
A) had defended themselves successfully against aphids.
B) were developed in the laboratory using bioengineering.
C) had no exposure to caterpillars before being seeded with aphids.
D) showed no evidence of either bacterial or viral infection.
232. The researchers investigated the birds’ interest in novel objects or “object neophilia,” which is defined in the paper as
“exploration in which investigation is elicited by an object’s novelty.” To do this a bunch of threads were placed on a perch
within the cage. The time taken for the birds to approach the threads within one body length and to touch them was recorded
over a one-hour period. It was found that the aggressive red-headed birds showed less interest in novel objects than did black-
headed birds. The difference is not as striking but was statistically significant nonetheless.
These experiments were repeated after a two-month interval and showed that different birds differed in their responses but the
responses of individual birds were consistent over time.
It can reasonably be inferred from the text that it would be atypical for an individual red-headed finch to
A) resume feeding relatively slowly after a predator display one week and resume just as slowly the next.
B) approach novel objects without hesitation one week but entirely avoid them the next.
C) return to feeding after a predator display consistently faster than another red-headed finch.
D) display aggression one week and continue to display aggression the next.
233. The researches suspected that feeding by one group of insects,such as chewing caterpillars, might affect the plant’s ability to
fight off another group, like aphids. To test this idea, the researchers grew corn seedlings of a common variety, called B73, and
exposed some to caterpillars. They then seeded them with aphids and counted the number of offspring that the aphids
produced on pristine plants, compared to previously nibbled ones. The aphids consistently produced more offspring on corn
that had been pre-chewed by caterpillars.
But when the researchers tested other corn varieties, individual results would vary. They repeated the experiment with 17
different lines of corn from around the world. Like B73, some varieties supported more aphid offspring after a caterpillar
feeding, while the pre-feeding reduced the number of aphids or had no effect on other varieties.
Which choice is an accurate statement, regarding the relationship between the researchers initial hypothesis and the results of
the experiment discussed in the passage?
A) The results of the experiment fully confirmed the initial hypothesis
B) The results of the experiment partially supported the initial hypothesis.
C) The results of the experiment suggested that the initial hypothesis was unclear.
D) The results of the experiment definitively disproved the initial hypothesis
234. The results showed that the range of striped species overlaps with where biting flies are most active— regardless of species
and where the stripes occur on the body according to the study.
Brenda Larison, a biologist at the University of California, Los Angeles, who studies stripes in plains zebras, said the new
study’s approach is “broad brush,” and that more specific research may be needed.
That’s why “the story is likely to be much more complex, and this is unlikely to be the last word on the subject,” said Larison.
In describing the new study’s approach as “broad brush”, Larison suggests that the study
A) produced results that are relevant to an array of scientific disciplines.

58
https://hocsat.com/
B) revealed that further research will require a specialized focus.
C) raised theoretical questions that proved unanswerable.
D) collected far more data than could be analyzed properly.
235. The scientists showed participants word lists, then removed the lists and tested the subjects on words that had and hadn’t been
included. Each list invoked a so-called critical lure—a word commonly associated with the words on the list, but that did not
actually appear on the list. The word sleep, for example, might be falsely remembered as appearing on a list that included the
words pillow, duvet and nap. All of the participants in both groups fell for the lures, with at least eight such errors per person.
According to the text, one characteristic of the word lists used in the study was that each list
A) appeared in conjunction with related visual images.
B) consisted of words intended to evoke emotional reactions.
C) included words related to a central theme or topic.
D) made use of relatively straightforward words.
236. The team also found five genes in domestic cats that influence the migration of neural crest cells, stem cells in the developing
embryo that affect everything from skull shape to coat color. This supports a recent proposal that such cells may be the master
control switches of domestication, explaining why domestic animals share _______________, such as smaller brains and
certain pigmentation patterns.
Which choice completes the text with the most logical and precise word or phrase?
A) widespread mannerisms.
B) inferior qualities.
C) familiar habits.
D) similar features.
237. The term GMO itself is a catchall that encompasses a wide range of products developed through a variety of means, each with
its own risks and benefits. There are GMOs that have led to large reductions in the use of pesticides, for example, and there are
GMOs that have made herbicide use skyrocket. The broad brush fails when labeling the developers of GM technology:
Commercial giants of the agrochemical pesticide industry have developed GMOs, but so have academic scientists funded by
nonprofits for the public sector.
“A technology like GM crops is neither good nor bad,” Qaim says. “Talking about the impact of GMOs is way too broad.”
In context, the italicization of the word “the” serves to convey which meaning?
A) That there is one kind of GMO that governments need to be especially concerned about
B) That GMOs are the sole technology that can address the world’s hunger problems
C) That the notion that GMOs could have only a single effect is simplistic
D) That GMOs are the most significant issue that science is grappling with today
238. The United States Government has solved this problem some time since, for its own purposes, and provides a working model
of how to do it, and what is to be gained from it. Each year the Government buys some $300,000,000 of supplies and
equipment—ranging all the way from thumbtacks to dredging machines; from baseballs to battleships. Nearly every kind of
thing the ___________ consumer buys, the Government at Washington buys—though in not such great variety—foodstuffs,
textiles, clothing, furniture, building materials, office supplies, sporting goods, toilet articles.
Which choice completes the text with the most logical and precise word or phrase?
A) collective.
B) widespread.
C) approximate.
D) ordinary.
239. The United States Government has solved this problem some time since, for its own purposes, and provides a working model
of how to do it, and what is to be gained from it. Each year the Government buys some $300,000,000 of supplies and
equipment—ranging all the way from thumbtacks to dredging machines; from baseballs to battleships. Nearly every kind of

59
https://hocsat.com/
thing the ___________ consumer buys, the Government at Washington buys—though in not such great variety—foodstuffs,
textiles, clothing, furniture, building materials, office supplies, sporting goods, toilet articles ...
Which choice completes the text with the most logical and precise word or phrase?
A) collective.
B) widespread.
C) approximate.
D) ordinary.
240. The variation they saw is likely due to the evolutionary history of the different corn varieties. Aphids tend to be more common
in temperate areas, such as the Midwest, where they spread barely yellow dwarf virus and cereal yellow dwarf virus, while
caterpillars are a larger problem in tropical areas. Different varieties likely arose from breeding programs aimed at fighting the
threats that corn faces in different environments.
According to the text, the different varieties of corn might have arisen from a need for corn producers to
A) guard against local dangers.
B) increase nutritional value.
C) standardize the quality of production.
D) record evolutionary tendencies.
241. Then, in 1999, the group looked more closely at the characteristics of the slow fibers. They named these “low-threshold”
nerves “C-tactile,” or CT fibers, said Olausson, because of their “exquisite sensitivity” to slow, gentle tactile stimulation, but
unresponsiveness to noxious stimuli like pinpricks.
But why exactly humans might have such fibers, which respond only to a narrow range of rather subtle stimuli, was initially
mystifying. Unlike other types of sensory nerves, CT fibers could be found only in hairy human skin—such as the forearm and
thigh. No amount of gentle stroking of hairless skin, such as the palms and soles of the feet, prompted similar activity
signatures. Olausson and his colleagues decided that these fibers must be conveying a different dimension of sensory
information than fast-conducting fibers.
It can reasonably be inferred that one of the intended goals of the 1999 experiment was to determine the
A) precise nature of sensations that CT fibers can convey.
B) relationship between body hair and CT fiber function.
C) role played by CT fibers in the perception of pain.
D) effect of microneurography on CT fiber signaling.
242. There might, though, be another way of getting at P. fulva. This would be to craft a type of virus, known as a bacteriophage,
specific to the bug—an approach already being investigated for the treatment of human illness caused by a different species of
Pseudomonas.
In practice, more than one type of phage would probably be needed, for if P. fulva were knocked out, another caffeine-
consuming bacterium in the beetle’s gut might end up replacing it. But, regardless of the details, this study has introduced a
novel way of thinking about pest control.
The author’s discussion best supports which statement about P. vulva?
A) It is probably more resistant to antibiotics than are other species of caffeine-consuming bacteria that live in the guts of
coffee-berry borers.
B) It is probably not the only species of bacteria that can live in the guts of coffee—berry borers and digest or detoxify
caffeine.
C) It can probably be killed by the bacteriophage that has been developed to treat illnesses caused by another species of
Pseudomonas.
D) It can probably survive only in the guts of the various insect species that regularly ingest caffeine.
243. They were similar enough in build to draw from a single pile of clothes. Their complexions, a light coppery compound derived
from their parents, were identical. Their double-jointed fingers, the sharp cut of their features, the wavy texture of their hair.

60
https://hocsat.com/
Subhash wondered if his placid ___________ was regarded as a lack of inventiveness, perhaps even a failing, in his parents'
eyes.
Which choice completes the text with the most logical and precise word or phrase?
A) temperament.
B) influence.
C) environment.
D) shape.
244. Thirty—two hours after Hattie and her mother and sisters crept through the Georgin woods to the train station, thirty-two
hours on hard seats in the commotion ofthe Negro car, Hattie was startled from a light sleep by the train conductor’s bellow,
‘Broad Street Station, Philadelphia!‘ Hattie clambered from the train, her skirt still hemmed with Geoigia mud, the dream of
Philadelphla round as a marble in her mouth and the fear of it a needle in her chest.
The phrases “round as a marble in her mouth" and “a needle in her chest” have the primary effect of
A) dramatizing the fleeting nature of an intense emotion.
B) underscoring the discrepancy between reality and subjective perception,
C) demonstrating the reconciliation of seemingly contradictory states of mind.
D) conveying the idea that anticipation encompasses both hopefulness and dread.
245. This passage is adapted from Jane Austen, Persuasion. Originally published in 1818. Anne is one of three daughters of Sir
Walte EIIiot, a widower. Lady Russell is a longtime family friend.
Anne Elliot, with all her claims of birth, beauty and mind, to throw herself away at nineteen; involve herself at nineteen in an
engagement with a young man, who had nothing but himself to recommend him, and no hopes of attaining affluence, but in
the chances of a most uncertain profession, and no connexions to secure even his farther rise in that profession would be,
indeed, a throwing away, which she grieved to think of! Anne Elliot, so young; known to so few to be snatched off by a
stranger without alliance or fortune or rather sunk by him into a state of most wearing, anxious, youth-killing dependance! It
must not be, if by any fair interference of friendship, any representations from one who had almost a mother's love, and
mother's rights it would be prevented.
Regarding Annes relationship with Captain Wentworth, it can reasonably be inferred that Lady Russell had convinced herself
that
A) the couple did not know the true meaning of love.
B) she had a compelling obligation to intervene.
C) she was the only one who could influence Anne.
D) Anne should marry someone raised in Somersetshire.
246. This passage is adapted from Mary Helen Stefaniak, The Cailiffs of Baghdad, Georgia, ©2010 by Mary Helen Stefaniak.
Narrator Gladys Cailiff is eleven years old in 1938 when a new, well-traveled young schoolteacher, Miss Grace Spivey, turns a
small Georgia town upside down.
Miss Grace Spivey arrived in Threestep, Georgia, in August 1938. She stepped off the train wearing a pair of thick-soled boots
suitable for hiking, a navy blue dress, and a little white tam that rode the waves of her red hair at a gravity-defying angle.
In the text, Threestep is mainly presented as a
A) summer retreat for vacationers.
B) small rural town.
C) town that is home to a prominent university.
D) comfortable suburb.
247. This passage is adapted from Theodore Dreiser, Sister Carrie. Originally published in 1900.
It was not often that she came to the play stirred to her heart’s core by actualities. To-day a low song of longing had been set
singing in her heart by the finery, the merriment, the beauty she had seen. Oh, these women who had passed her by, hundreds
and hundreds strong, who were they? Whence came the rich, elegant dresses, the astonishingly coloured buttons, the knick-

61
https://hocsat.com/
knacks of silver and gold? Where were these lovely creatures housed? Amid what elegancies of carved furniture, decorated
walls, elaborate tapestries did they move? Where were their rich apartments, loaded with all that money could provide? In
what stables champed these sleek, nervous horses and rested the gorgeous carriages? Where lounged the richly groomed
footmen? Oh, the mansions, the lights, the perfume, the loaded boudoirs and tables! New York must be filled with such
bowers, or the beautiful, insolent, supercilious creatures could not be. Some hothouses held them. It ached her to know that she
was not one of them—that, alas, she had dreamed a dream and it had not come true. She wondered at her own solitude these
two years past—her indifference to the fact that she had never achieved what she had expected.
Throughout the text, the narrator provides insight into Carrie’s character mainly by
A) explaining how Carrie is similar to a character she sees in a play.
B) developing a contrast between how Carrie perceives herself and how she is perceived by others.
C) comparing Carrie’s actual appearance to her perceptions of her appearance.
D) juxtaposing Carrie’s perceptions of the city and her impressions at the theater.
248. This passage is adapted from Theodore Dreiser, Sister Carrie. Originally published in 1900.
It was not often that she came to the play stirred to her heart’s core by actualities. To-day a low song of longing had been set
singing in her heart by the finery, the merriment, the beauty she had seen. Oh, these women who had passed her by, hundreds
and hundreds strong, who were they? Whence came the rich, elegant dresses, the astonishingly coloured buttons, the knick-
knacks of silver and gold? Where were these lovely creatures housed? Amid what elegancies of carved furniture, decorated
walls, elaborate tapestries did they move? Where were their rich apartments, loaded with all that money could provide? In
what stables champed these sleek, nervous horses and rested the gorgeous carriages? Where lounged the richly groomed
footmen? Oh, the mansions, the lights, the perfume, the loaded boudoirs and tables! New York must be filled with such
bowers, or the beautiful, insolent, supercilious creatures could not be. Some hothouses held them. It ached her to know that she
was not one of them—that, alas, she had dreamed a dream and it had not come true. She wondered at her own solitude these
two years past—her indifference to the fact that she had never achieved what she had expected.
What is the main purpose of the series of questions in the text?
A) reveal Carrie’s ignorance of the women’s private struggles.
B) capture the scene’s authenticity by providing detailed descriptions of the women.
C) emphasize the narrator’s contempt for the women’s displays of affluence.
D) develop the narrator’s portrayal of Carrie’s intense admiration of privileged women.
249. This text is adapted from Gail Tsukiyama, Night of Many Dreams. @1998 by Gail Tsukiyama. Emma has sailed from Hong
Kong to the United States in 1950 to attend college.
From the crowded wharf, Sergei turned onto a street he called Columbus. “Like the explorer,” he said, slowing down as he
peeked at her in his rearview mirror. “And this is North Beach, where all the Italians live and eat.”
Emma looked away from the mirror. “From Italy?”
He nodded. “At one time.”
“And where do all the Russians live and eat?” “Wherever we can,” Sergei answered with a laugh. He drove several blocks,
then turned right. “This is Washington Street. Just remember, the first American president.” Then he made another right turn
onto Grant Avenue. “We are now in heart of Chinatown!” His thick eyebrows flashed upward as he again caught her eyes in
the mirror.
Sergei’s references to “the explorer” and “the first American president” serve mainly to
A) reveal his passion for learning historical trivia.
B) show his desire to help Emma make connections.
C) introduce a serious side to his otherwise jovial nature.
D) portray his need to impress Emma with his knowledge.
250. This text is adapted from Gail Tsukiyama, Night of Many Dreams. @1998 by Gail Tsukiyama. Emma has sailed from Hong
Kong to the United States in 1950 to attend college.

62
https://hocsat.com/
Not quite three weeks after leaving Hong Kong, Emma changed into a cotton cheungsam and finally set foot on Pier 19 in San
Francisco. Emma waited as the passengers disembarked and the crowd dissipated. What would become of her? Alone on the
pier, she listened for the voices of Mah-mee and Auntie Go telling her what to do, but they seemed lost across the ocean.
Emma took a deep breath and thought of her sister Joan’s advice: What would Lauren Bacall do?
In saying that the voices of Mah-mee and Auntie Go seem “lost”, the narrator most likely means that they seem
A) strange and unfamiliar.
B) desperate and unsure.
C) distant and irretrievable.
D) unappreciated and misjudged.
251. This text is adapted from Rachel Ehrenberg, “GMOs under Scrutiny.” @2016 by Society for Science & the Public.
The diversity of engineering processes and the products that result will probably continue to grow. For example, the relatively
new CRISPR technology, which allows for superprecise gene editing, may soon become a GMO tool of choice. But generally
speaking, the technologies behind GMOs are decades old. And despite fears of unknown risks, GMOs have been studied
extensively.
The author cites CRISPR as an example of
A) recent technological innovations related to GMOs.
B) a GMO technique whose impact has been extensively studied.
C) an especially risky method of modifying genes.
D) less controversial approaches to increasing crop yields.
252. Those who would have this Nation enter upon a career of empire must consider, not only the effect of imperialism on the
Filipinos, but they must also ________ its effects upon our own nation. We cannot repudiate the principle of self-government
in the Philippines without weakening that principle here.
Which choice completes the text with the most logical and precise word or phrase?
A) evaluate.
B) design.
C) assume.
D) multiply.
253. Three-spined sticklebacks swim in shallow waters along the coasts of Europe and North America. As with all evolutionary
tales, the stickleback story begins with variation. Some sticklebacks invest more in defensive weaponry than others, resulting
in fish-to-fish differences in the length of pelvic spines and in the size and number of body-armor plates. Not surprisingly, this
variation in weapon size influences fish survival. Long spines make sticklebacks difhcult to swallow, and armor plates protect
sticklebacks whenever predatory fish make the mistake of trying to bite them, Almost 90 percent of attacks on sticklebacks
fail. But before spitting them out, predators chew sticklebacks rather harshly. A sticklebacks armor plates act like shields,
reducing the extent of injuries from these bites.
According to the text, stickleback armor plates serve to
A) prevent damage to the stickleback from a predator's teeth.
B) make sticklebacks difficult to swallow.
C) help sticklebacks be more aggressive predators.
D) allow sticklebacks to survive in freshwater environments.
254. To conduct the study, a team led by psychologist Lawrence Patihis of the University of California, Irvine, recruited a sample
group of people all of approximately the same age and divided them into two subgroups: those with ordinary memory and
those with what is known as highly superior autobiographical memory (HSAM). You’ve met people like that before, and they
can be downright eerie. They’re the ones who can tell you the ________ date on which particular events happened—whether
in their own lives or in the news—as well as all manner of minute additional details surrounding the event that most people
would forget the second they happened.

63
https://hocsat.com/
Which choice completes the text with the most logical and precise word or phrase?
A) precise.
B) rigorous.
C) honest.
D) distinct.
255. To test the birds’ willingness to take risks, the researchers deprived the birds of food for one hour before the birds’ feeder was
replaced. After the birds had calmly begun to feed, a silhouette of an avian predator was moved up and down in front of the
cage to scare the birds from the feeder. The time it took for them to return to the feeder was taken as a measure of their
willingness to take risks. Birds that returned quickly were considered to be greater risk takers than those that were more
cautious.
This time the results were surprising. Red-headed birds were considerably more cautious than those with black heads at
returning to the feeder after a “predator” had been introduced. They took on average four times longer to begin feeding again
than the less aggressive black-headed birds.
Information in the text indicates that the purpose of the quotation marks around the word “predator” is to
A) imply that the predator was not notably dangerous.
B) indicate that the predator was actually a simulation.
C) show that the predator was used in multiple experiments.
D) reinforce the disruptive nature of the predator’s presence.
256. Today, sticklebacks in many lakes lack defensive weapons. Dolph Schluter and his students at the University of British
Columbia found that lake habitats have far fewer predators than marine habitats, and this appears to relax the pattern of natural
selection for larger plates and longer spines. With fewer predators, lake fish benefit less from large weapons than marine fish.
Armor also costs more in lakes than it does in the ocean. Low freshwater concentrations of the ions necessary for bone growth
mean that fish pay a higher price for mineralizing bony plates in lakes. Unarmed sticklebacks are larger as juveniles and begin
breeding sooner than their armed counterparts. In freshwater, it appears, the costs of long spines and large plates are steeper
than the benefits they provide.
As presented in the text, the research of Schluter and his students implies that in freshwater environments, it is better for
sticklebacks to
A) allocate scarce resources to basic skeletal formation rather than armor.
B) use armor to prey on other fish rather then defend themselves from predators.
C) grow longer spines than sticklebacks in ocean habitats.
D) begin reproducing later than they would in ocean environments.
257. Triplett recruited forty children, ages eight to thirteen, to complete his study in 1897. He measured how quickly the students
could wind a fishing reel so that a small flag attached to the line traveled a distance of sixteen meters. The task was
__________ but novel, and none of the children had played with fishing rods before the experiment. They performed the task
both alone and in the presence of other children, and Triplett noticed that they wound the reels faster in the presence of others.
He concluded that an audience enables people to “liberate latent energy” not normally available when they perform alone.
Which choice complete the text with the most logical and precise word or phrase?
A) Easy; straightforward
B) Mindless; sincere
C) Effortless; humble
D) Innocent; uncomplicated
258. Triplett recruited forty children, ages eight to thirteen, to complete his study in 1897. He measured how quickly the students
could wind a fishing reel so that a small flag attached to the line traveled a distance of sixteen meters. The task was simple but
novel, and none of the children had played with fishing rods before the experiment. They performed the task both alone and in
the presence of other children, and Triplett noticed that they wound the reels faster in the presence of others. He concluded that

64
https://hocsat.com/
an audience enables people to “liberate latent energy” not normally available when they perform alone.
As presented in the text, Triplett would most likely agree that human beings
A) are capable of ultimately mastering almost any task with which they are presented.
B) exhibit a general tendency to avoid attempting difficult tasks when they are alone.
C) have a strong desire to cooperate with other members of a group.
D) possess abilities that they are not always able to exploit.
259. Triplett recruited forty children, ages eight to thirteen, to complete his study in 1897. He measured how quickly the students
could wind a fishing reel so that a small flag attached to the line traveled a distance of sixteen meters. The task was simple but
novel, and none of the children had played with fishing rods before the experiment. They performed the task both alone and in
the presence of other children, and Triplett noticed that they wound the reels faster in the presence of others. He concluded that
an audience enables people to “liberate latent energy” not normally available when they perform alone.
Based on the text, the design of Triplett’s fishing- reel experiment most likely ruled out which potential objection to his
findings?
A) Improvements in the subjects’ performance after being asked to perform the task in front of an audience may reflect the
subjects’ increasing competence at the task.
B) Differences between the subjects’ performance with an audience and their performance without an audience may be
influenced by the subjects’ prior experience with the task.
C) Variations in performance among the subjects under either audience condition may be attributed to variations in the
subjects’ ages and physical development.
D) Changes in the subjects’ performance after being put in front of an audience may result from some subjects observing and
imitating the performance of more highly skilled subjects.
260. Udayan ran down the long wooden plank temporarily set up to get from the door to the street. Halfway across the plank he lost
his balance, the evidence of his path forming impressions of the soles of his feet, tapering like an hourglass at the center, the
pads of the toes disconnected. When the surface had dried, and the impressions left by Udayan’s feet were permanent. The
only way to repair the flaw was to apply another layer.
His father said, "Leave it be." Not for the expense or effort involved, but because he believed it was wrong to erase steps that
his son had taken. And so the imperfection became a mark of distinction about their home. Something visitors noticed, the first
family anecdote that was told.
The narrator suggests that as time passed, the incident involving the footprints took on a significance most like that of a
A) familiar legend whose exact origin cannot be identified.
B) historical narrative that suggests how one should behave in the present.
C) personal memory that one recounts to a new acquaintance.
D) cautionary tale that one tells to discourage careless behavior.
261. United States politician: Although the amount of United States goods shipped to Mexico doubled in the year after tariffs on
trade between the two countries were reduced, it does not follow that the reduction in tariffs caused the sales of United States
goods to companies and consumers in Mexico to double that year, because ___________
Which of the following, if true, most logically completes the politician's argument?
(A) many of the United States companies that produced goods that year had competitors based in Mexico that had long
produced the saM0 kind of goods
(B) most of the increase in goods shipped by United States companies to Mexico was in parts shipped to the companies’ newly
relocated subsidiaries for assembly and subsequent shipment back to the United States
(C) marketing goods to a previously unavailable group of consumers is most successful when advertising specifically targets
those consumers, but developing such advertising often takes longer than a year
262. We began an ambitious program at visible and near-infrared wavelengths as a natural successor to HUDF: the Cosmic
Assembly Near-infrared Deep Extragalactic Legacy Survey (CANDELS), pronounced “candles.” We designed CANDELS

65
https://hocsat.com/
primarily to document the first one-third of galaxy evolution. The program also would enable astronomers to search for the
most distant Type Ia supernovae—exploding white dwarf stars that are the best-known standard candles for measuring the
universe’s recent expansion rate. CANDELS could thus test whether Type Ia supernovae are also a valid yardstick for the
early universe.
According to the text, Type Ia supernovae may be useful because they could allow astronomers to
A) determine the expansion rate of the universe soon after the big bang.
B) more accurately calibrate other methods of measuring distance.
C) better understand what causes white dwarf stars to explode.
D) catalog the earliest known galaxies according to their physical properties.
263. When ___________ to state his age, Ranga would generally reply, “Fifty, sixty or eighty.” You might
change your tactics and inquire, “"How long have you been at this job?”
“Which job?”
“Carrying that grinding wheel around and sharpening knives.”
“Not only knives, but also scythes, clippers and every kind of peeler and cutter in your kitchen, also bread knives, even
butcher’s hatchets in those days when I carried the big grindstone; in those days I could even sharpen a maharaja’s sword” (a
favorite fantasy of his was that if armies employed swords he
could become a millionaire).
Which choice completes the text with the most logical and precise word or phrase?
A) pressed
B) occupied
C) rushed
D) afflicted
264. When mercury-vapor streetlights are used in areas inhabited by insect-eating bats, the bats feed almost exclusively around the
lights, because the lights attract flying insects. In Greeiiville, the mercury-vapot streetlights are about to be replaced with
energy- saving sodium streetlights, which do not attract insects. This change is likely to result in a drop in the population of
insect-eating hats in Greenville, since_______________
Which of the following most logically completes the argument below?
A. the bats do not begin to hunt until after sundown
B the bats are unlihely to feed on insects that do not fly
C. the highway department will be able to replace mercury-vapor streetlights with sodium streetlights within a relatively short
time and without disrupting the continuity of lighting at the locations of the streetlights
D. in the absence of local concentrations of the flying insects on which bats feed, the bats expend much more energy on
hunting for food, requiring much larger quantities of insects to sustain each bat
265. When officials in Tannersburg released their plan to widen the city's main roads, environmentalists protested that widened
roads would attract more traffic and lead to increased air pollution. In response, city officials pointed out that today's pollution-
control devices are at their most effective in vehicles traveling at higher speeds and that widening roads would increase the
average speed of traftc. However, this effect can hardly be expected to offset the effect pointed out by environmentalists, since
____________
Which of the following most logically completes the argument?
(A) increases in traffic volume generally produce decreases in the average speed of traffic unless roads are widened
(BI several of the roads that are slated for widening will have to be closed temporariIy'whiIe construction is underway
(C) most of the air pollution generated by urban traffic comes from vehicles that do not have functioning pollution-control
devices
(D) the newly widened roads will not have increased traffic volume if the roads that must be used to reach them are inadequate

66
https://hocsat.com/
266. When pressed to state his age, Ranga would generally reply, “Fifty, sixty or eighty.” You might change your tactics and
inquire, “How long have you been at this job?”
“Which job?”
“Carrying that grinding wheel around and sharpening knives.”
“Not only knives, but also scythes, clippers and every kind of peeler and cutter in your kitchen, also bread knives, even
butcher’s hatchets in those days when I carried the big grindstone; in those days I could even sharpen a maharaja’s sword” (a
favorite fantasy of his was that if armies employed swords he could become a millionaire).
The text indicates that compared to the past, Ranga now
A) can afford to be charitable to others.
B) indulges more often in daydreams.
C) sharpens fewer types of blades.
D) can attract a younger group of customers.
267. When ruling on big, controversial cases, the justices split fairly reliably along party lines dedicated by their appointing
presidents. It wasn't always this way. Until 1937, party simply wasn't factor in high court decisions. Only in recent decades
have party politics infiltrated the marble halls of the Supreme Court, and only in the past few years have they become the best
predictor of its major ruling. The Supreme Court has never divided along partisan lines as neatly as it does today.
The author states that over the course of recent decades, the decisions of Supreme Court justices increasingly correlate with
A) an emerging national consensus on free-speech issues.
B) shifts in the tone of political discussion in the United States.
C) personal experiences in the justices’ lives outside of the courtroom.
D) the political affiliation of the presidents who appointed the justices.
268. Zabrocki’s experience working in many different capacities in the music industry is a common one: a 2011 survey of 5,371
performers conducted by the nonprofit Future of Music Coalition demonstrated the vast majority of respondents earned
revenue in multiple ways. Researchers identified eight types of work that musicians commonly do to earn revenue, including
composing original music, writing lyrics, and performing live. Only 983 respondents —roughly 18 percent—said that all of
their revenue came from a single type of work. _________

Which choice most effectively uses data from the graph to complete the text?
A) More respondents reported having four revenue streams than reported having five.
B) Most reported having two or three revenue streams, and many reported having even more.
C) About half as many respondents reported having four revenue streams as reported having two.
D) Very few respondents derived income from all of the revenue sources included in the survey.

67
https://hocsat.com/
269. When the first European earthworms arrived, they began doing what they always do: munching, mixing and moving. Some
plant litter earthworms munched through the forest floor and its fungi and bacteria. Burrowing species, like the common
nightcrawler, pulled leaf litter down into their holes to finish munching and mixing. Slowly, earthworms destroyed the duff on
which wildflowers, understory shrubs and tree seedlings depended.
The text implies that the feeding behavior of earthworms can vary according to the
A) species of earthworm.
B) earthworms’ proximity to predators.
C) size of the earthworm population.
D) weather conditions earthworms encounter.
270. When they were old enough, when they were permitted to leave the house, they were told not to lose sight of one another.
Together they wandered down the winding lanes of the enclave, behind the ponds and across the lowland, to the playing field
where they sometimes met up with other boys. They went to the mosque at the corner, to sit on the cool of its marble steps,
sometimes listening to a football game on someone’s radio, the guardian of the mosque never minding.
Eventually they were allowed to leave the enclave, and to enter the greater city. To walk as far as their legs would carry them,
to board trams and busses by themselves. Still the mosque on the corner, a
place of worship for those of a separate faith, oriented their daily comings and goings.
The text indicates that as the brothers age, they are allowed greater freedom to
A) explore the world outside their home.
B) assist in household chores.
C) spend time apart from one another.
D) pursue work in the film industry.
271. While most sticklebacks live in the ocean where predators are __________, some inhabit freshwater lakes, and here their
evolutionary story is different. Ocean levels fluctuate greatly over time, and during periods of high water fish spill into inland
reservoirs, where they end up trapped as the water recedes. Inland fish experience very different patterns of selection from
their marine ancestors, and in lake after lake, their weapons have changed as populations adapted to their new locales.
Which choice completes the text with the most logical and precise word or phrase?
A) largely inferior.
B) equally shared.
C) generally plain.
D) frequently encountered.
272. While the total enrollment of public elementary and secondary schools in Sondland is one percent higher this academic year
than last academic year, the number of teachers there increased by three percent. Thus, the Sondland Education Commission's
prediction of a teacher shortage as early as next academic year is unfounded.
Which of the following, if true, most seriously weakens the claim that the prediction of a teacher shortage as early as next
academic year is unfounded?
(A) Funding for public elementary schools in Sondland is expected to increase over the n0xt ten years.
(B) Average salaries for Sondland's teachers increased at the rate of inflation from last academic year to this academic year.
(C) A new law has mandated that there be ten percent more teachers per pupil in Sondland's public schools next academic year
than there were this academic year.
(D) In the past, increases in enrollments in public elementary and secondary schools in Sondland have generally been smaller
than increases in the number of teachers.
273. Within the earth's core, which is iron, pressure increases with depth. Because the temperature at which iron melts increases
with pressure, the inner core is solid and the outer core is molten. Physicists can determine the melting temperature of iron at
any given pressure and the pressure for any given depth in the earth. Therefore, the actual temperature at the boundary of the
earth's outer and inner cores—the melting temperature of iron theie—can be determined, since ___________

68
https://hocsat.com/
Which of the following most logically completes the argument?
(A) the depth beneath the earth's surface of the boundary between the outer and inner cores is known
(B) some of the heat from the earth's core flows to the surface of the earth
(C) pressures within the earth's outer core are much greater than pressures above the outer core
(D) nowhere in the earth's core can the temperature be measured directly
274. Yanping Tu of the University of Chicago and Dilip Soman of the University of Toronto examined how individuals go about
both thinking about and completing tasks. Previous studies have shown that such activity progresses through four distinct
phases: pre-decision, post-decision (but pre-action), action and review. It is thought that what motivates the shift from the
decision-making stages to the doing-
something stage is a change in mindset.
Human beings are a deliberative sort, weighing the pros and cons of future actions and remaining open to other ideas and
influences. However, once a decision is taken, the mind becomes more “implemental” and focuses on the task at hand. "The
mindset towards 'where can I get a sandwich’,” explains Ms. Tu, “is more implemental than the mindset towards 'should I get a
sandwich or not?”
The text implies that just before people enter the action-taking phase, their thinking is characterized by a concern for the
A) practical planning involved in completing a task.
B) benefits awarded to those who undertake a task.
C) speed with which a task can be completed.
D) goals of the person who set a task's deadline.
275. You [United States] are young, have the perfectest of dialects, a free press, a free government, the world forwarding its best to
be with you. As justice has been strictly done to you, from this hour do strict justice to yourself. Strangle the singers who will
not sing you loud and strong. Open the doors of The West. Call for new great masters to comprehend new arts, new
perfections, new wants. Submit to the most robust bard till he remedy your barrenness. Then you will not need to adopt the
heirs of others; you will have true heirs, begotten of yourself, blooded with your own blood.
What choice best states the central claim of the text?
A) Americans are unfortunately resistant to the idea of creating a literature that is new and original.
B) the variety of styles in American literature makes it superior to that of Europe.
C) the creation of original American literature is both needed and overdue.
D) American literature will eventually influence the literary development of other nations.
276. You find upon the statute books under Washington and the early Presidents provisions of law showing that in the southwestern
territories the right to hold slaves was clearly implied or ______________, while in the northwest territories it was prohibited.
The only conclusion that can be fairly and honestly drawn from that legislation is that it was the policy of the fathers of the
republic to prescribe a line of demarcation between free territories and slaveholding territories by a natural or a geographical
line, being sure to make that line correspond, as near as might be, to the laws of climate, of production, and probably of all
those other causes that would control the institution and make it either desirable or undesirable to the people inhabiting the
respective territories.
Which choice completes the text with the most logical and precise word or phrase?
A) formally acknowledged.
B) publicly commended.
C) plainly confessed.
D) accurately recalled.
277. Young Tchartkoff was an artist of talent, which promised great things: his work gave evidence of observation, thought, and a
strong inclination to approach nearer to nature.
“Look here, my friend,” his professor said to him more than once, “you have talent; it will be a shame if you waste it: but you
are impatient; you have but to be attracted by anything, to fall in love with it, you become engrossed with it, and all else goes

69
https://hocsat.com/
for nothing, and you won’t even look at it. See to it that you do not become a fashionable artist.
The first paragraph serves mainly to establish the
A) ironic outlook of the narrator.
B) central conflict depicted in the passage.
C) main character’s defining artistic traits.
D) relationship between two characters.
278. Young Tchartkoff was an artist of talent, which promised great things: his work gave evidence of observation, thought, and a
strong inclination to approach nearer to nature. “Look here, my friend,” his professor said to him more than once, “you have
talent; it will be a shame if you waste it: but you are impatient; you have but to be attracted by anything, to fall in love with it,
you become engrossed with it, and all else goes for nothing, and you won’t even look at it. See to it that you do not become a
___________ artist.
Which choice completes the text with the most logical and precise word or phrase?
A) stylish.
B) trendy.
C) modern.
D) conventional.
279. The 2012 study found that local attendees to arts events spent _________ the cost of admission per person per event, while
nonlocal attendees spent an average of $39.96 per person. The largest portion of spending for both local and nonlocal attendees
was on meals, snacks, and other refreshments, which brought in an average of $13.14 per person per event.

Which choice most effectively uses data from the graph to complete the text?
A) an average of $24.60 on top of
B) an average of $17.42 in addition to
C) an average of $17.42 including
D) $11.16 less than the average spending of all attendees, including

70
https://hocsat.com/
280. Zajonc and a team of researchers devised two small athletic tasks that required the cockroaches to scuttle from a brightly lit
area in a small box to a more appealing darker compartment. Some of the cockroaches completed a simpler task, in which they
ran along a straight runway from the glare of the box to the darkened goal compartment. The remaining cockroaches
completed a more difficult task, traversing a more complex maze before they could escape the light. Some of the cockroaches
completed these tasks alone, but the researchers also built small audience box to force some of the athletic cockroaches to
compete in front of an audience of roach spectators. The cockroaches were much quicker to cover the straight runway when
watched by an audience, reaching the darkened goal compartment an average of twenty-three seconds more quickly when they
were performing before a crowd. But the cockroach athletes responded very differently to an audience when they were faced
with the complex maze, reaching the goal seventy- six seconds more quickly when they were alone. The same audience that
pushed the cockroaches to perform the simpler task more quickly also delayed them when the task was more complex
The author uses the terms “athletic cockroaches,” “roach spectators,” and “cockroach athletes” in the text most likely to
A) call attention to the similarity of the two experiments described.
B) stress the skepticism with which the author views the design of the experiment.
C) create a casual tone that offsets the seriousness of the work being done.
D) reinforce the idea that the observed cockroach behavior is comparable to that of humans.

71
https://hocsat.com/

You might also like